When Mrs Kline is 2 days postpartum The nurse would expect fundal height to be?

A 24-year-old nulligravida female presents for her annual examination. Her gynecologic history is remarkable for irregular menses, menstruating every 4 to 8 weeks. She would like a more reliable form of contraception (currently using condoms) and would like to have predictable menses, but is very concerned regarding weight gain with various contraceptive methods.

Question 15.1.1 How would you counsel her regarding weight changes and contraception?

A) Studies show there is no significant difference in weight gain of women initiating oral contraceptive pills (OCPs) versus placebo

B) Weight gain of 10 lb is expected during the first year of use with any type of OCPs

C) Weight gain of 10 lb is expected during the first year of use with monophasic pills but not with triphasic formulations

D) Weight loss of 10 lb is expected during the first year of use with Depo-Provera (medroxyprogesterone acetate)

Answer 15.1.1 The correct answer is "A." Studies have shown no significant weight gain with OCP use when compared with placebo. Trials have been conducted evaluating estrogen components of 20 to 50 μg, both monophasic and triphasic. There is no evidence to support the premise that triphasic formulations offer improvement in weight changes. Depo-Provera has variable effects on weight gain. Several studies have shown a weight gain of 3 to 6 kg in the first year of use; however, other studies have shown no difference in weight gain between Depo-Provera and placebo.

When Mrs Kline is 2 days postpartum The nurse would expect fundal height to be?
HELPFUL TIP:

Recall that in 2004 the FDA placed a "black box warning" on Depo-Provera for bone loss that "may not be reversible." However, subsequent studies showed bone gain after stopping Dep-Provera. The prudent thing would be to avoid Depo-Provera in women at high risk of osteoporosis.

Question 15.1.2 After reassuring her regarding the concerns of weight gain, you tell her about the additional potential benefit(s) of OCPs, which include:

A) Improvement in acne

B) Decreased dysmenorrhea

C) Decreased menstrual flow

D) Decreased risk of ovarian cancer

E) All of the above

Answer 15.1.2 The correct answer is "E." Besides these, additional potential benefits of OCP use include regulation and predictability of menses, decreased anemia, decreased hirsutism, and decreased risk of endometrial and colon cancers. (Note: the cancer risk reduction is based on epidemiologic studies, not randomized controlled trials.)

When Mrs Kline is 2 days postpartum The nurse would expect fundal height to be?
HELPFUL TIP:

After starting OCPs, patients should follow up within a few months for blood pressure checks, assuring compliance, etc.

After further discussion, she reports that she has headaches every 1 to 2 months. She has never been evaluated for migraines, but reports that her headaches are bilateral, posterior, throbbing, and relieved with sleep and over-the-counter medication. She denies associated aura, nausea, or focal neurologic changes.

Question 15.1.3 How would you counsel her regarding OCPs and headaches?

A) Headaches are an uncommon reason for discontinuation of OCPs

B) She should not use OCPs because they are contraindicated in anyone with headaches

C) She should use progestin-only pills

D) She can use OCPs, as it is hard to predict whether her headaches will be affected

Answer 15.1.3 The correct answer is "D." Although headache is a frequently cited reason for women to discontinue OCPs, there is no strong correlation between headache frequency and intensity for most women. There is no evidence that the type of progestin or amount of estrogen will alter the headaches, except in women with menstrual migraines. Among women with migraines, headaches improve, worsen, or are unchanged after initiation of OCPs (helpful, right?). There is an increased risk of stroke in women with a history of pseudotumor cerebri or migraines with aura or focal neurologic changes. Therefore, OCPs are CONTRAINDICATED in these specific subgroups of women.

When Mrs Kline is 2 days postpartum The nurse would expect fundal height to be?
HELPFUL TIP:

Additional contraindications to combination OCP use include any previous thromboembolic event or stroke, a history of estrogen-dependent tumor (e.g., some breast cancers), active liver disease, pregnancy (although accidental use of OCPs early in pregnancy has not been definitively linked to adverse outcomes), undiagnosed abnormal uterine bleeding, women older than 35 years who smoke (due to increased risk of cardiovascular disease), and first 3 weeks postpartum (2011 CDC recommendation based on increased venous thromboembolism risk in the immediate postpartum period).

The patient wants to know how long she should use a backup method of contraception after starting the combination estrogen/progesterone OCP.

Question 15.1.4 You tell her the following regarding using a backup method:

A) If she starts taking the pill during the first 5 days of her menstrual period, no backup method is needed

B) If she starts taking the pill greater than 7 days after the onset of her period, she needs a backup method for 7 days

C) She should use a backup method for 2 months after starting the pill regardless of when she started it

D) The OCP provides effective contraception immediately and no backup method is needed

Answer 15.1.4 The correct answer is "A." The old canard of requiring a month of backup contraception after starting OCPs, implants, etc. has fallen by the wayside. For combination estrogen/progesterone OCPs, for example, no secondary method of contraception is needed as long as the OCP is started within 7 days of the onset of the last menses. If the OCP is started greater than 7 days after the onset of the last menses, only 7 days of a secondary contraception method (e.g., condoms, diaphragm) is needed; combination OCPs work within 7 days of the start date. Please see Table 15-1 regarding CDC recommendations for backup contraception.

TABLE 15-1BACKUP CONTRACEPTION WHEN INITIATING METHODS

View Table||Download (.pdf)

TABLE 15-1 BACKUP CONTRACEPTION WHEN INITIATING METHODS

Contraceptive MethodBackup Method: Initiationa
Copper-containing IUD (Paraguard) None needed
Levonorgestrel-containing IUD 7 days; only needed if >7 days after starting menses
Implant 7 days; only needed if >5 days after starting menses
Injectable 7 days; only needed if >7 days after starting menses
Combined hormonal contraceptives 7 days; only needed if >5 days after starting menses
Progestin-only pill 2 days; only needed if >5 days after starting menses

When Mrs Kline is 2 days postpartum The nurse would expect fundal height to be?
HELPFUL TIP:

Rifampin, rifaximin and rifabutin are the only antibiotic that reduces the effectiveness of OCPs. Therefore, there is no need for a backup method when giving penicillin for strep throat, for example.

When Mrs Kline is 2 days postpartum The nurse would expect fundal height to be?
HELPFUL TIP:

The thinking about methods of contraception has changed in the last several years. The IUD is becoming increasingly popular for nulliparous teenagers. IUDs clearly decrease pregnancy in this group when compared to OCPs and do not seem to increase the risk of STDs (N Engl J Med. 2014;371:1316). Nor do they result in increased risky sexual activity.

The American Academy of Pediatrics (2014) recommends that a long-acting reversible contraceptive (LARC) be considered as a first-line contraceptive option in adolescents; options include implants and intrauterine devices (IUDs). LARCs can provide 3 to 10 years of contraception and are effective and safe forms of birth control. The use of condoms goes down with LARCs, however (JAMA Pediatr. 2016 Mar 14).

Eight months later, the patient calls to speak with your nurse regarding nausea and vomiting. Apparently, she decided to stop taking her OCP. Her last menstrual period was 10 weeks earlier, and she had a positive home pregnancy test 5 weeks ago. Over the last week, she has been vomiting once every day, at various times, but is nauseated throughout most of the day. She wants to know if there is anything else that is "safe" that she can do to decrease the nausea.

Question 15.1.5 What is your most appropriate response?

A) "This level of nausea and vomiting is abnormal and needs an immediate workup to rule out other pathology"

B) "This level of nausea and vomiting is very common, and there are several modifications and over-the-counter medications that are safe"

C) "This level of nausea and vomiting is very common; however, there are no medications that can be initiated in the first trimester"

D) "This level of nausea and vomiting is very common. Metoclopramide, promethazine, and ondansetron are our first-line therapies"

E) "Deal with it. You got yourself into this mess"

Answer 15.1.5 The correct answer is "B." Mild-to-moderate nausea and vomiting are very common in the first trimester of pregnancy, often improving by 16 weeks of gestation. Several modifications can improve symptoms, including small, frequent meals, avoiding fatty foods, and avoiding environmental triggers (perfumes, smoking, position changes, or certain movements). Over-the-counter remedies include ginger root or ale, the combination of vitamin B6 (10–25 mg three to four times daily) and doxylamine (10–12.5 mg three to four times daily). Vitamin B6 and doxylamine may be used separately. For those who do not respond, antiemetics including metoclopramide or promethazine should be considered (thus, "D" represents a next step, not a first step). Ondansetron would be third line due to slightly greater potential risk to the fetus. Up to 2% of the time, nausea and vomiting represent hyperemesis gravidarum, which involves weight loss of more than 5% of prepregnancy weight or dehydration and ketonuria.

When Mrs Kline is 2 days postpartum The nurse would expect fundal height to be?
HELPFUL TIP:

Diclegis (doxylamine succinate and pyridoxine hydrochloride) is now a FDA approved medication for nausea and vomiting of pregnancy. Patients should take 2 tablets at bedtime as a starting dose and may increase to 1 in the morning, 1 in the mid-afternoon, and 2 at nighttime, if needed. However, it is way expensive and has no advantage over the combination of OTC doxylamine and pyridoxine.

Question 15.1.6 If the patient had instead presented with 6 to 8 episodes of emesis daily, an 8-lb weight loss since her last menses, and a urine specific gravity of >1.030 and ketonuria, your workup should have included:

A) Quantitative β-human chorionic gonadotropin (β-hCG)

B) Serum electrolytes, BUN, and creatinine

C) Thyroid-stimulating hormone (TSH)

D) Pelvic ultrasound

E) All of the above

Answer 15.1.6 The correct answer is "E." As with any other severely nauseated and vomiting patient, it is reasonable to check for electrolyte imbalances. Severity of nausea and vomiting correlates with higher levels of hCG, as would be seen with a molar or twin pregnancy. Gestational trophoblastic disease, although rare, should be evaluated for with an hCG level. The ultrasound would confirm a twin pregnancy and could provide evidence of a molar pregnancy. TSH can exclude hyperthyroidism.

Fortunately, your patient's symptoms improved with dietary changes, vitamin B6, and doxylamine. She presents for her initial prenatal visit at 12 weeks of gestation.

Question 15.1.7 You offer her the routine prenatal tests at this visit, which include all of the following EXCEPT:

A) Syphilis testing

B) HIV testing

C) 1-hour glucose tolerance test

D) Blood type and antibody screen

E) Fetal nuchal translucency with maternal hCG and plasma-associated pregnancy protein A (PAPP-A)

Answer 15.1.7 The correct answer is "C." Diabetes screening with a 1-hour glucose tolerance test (50-g carbohydrate load with blood glucose obtained at 1 hour) is typically performed between 24 and 28 weeks of gestation. Patients with risk factors for gestational diabetes, which our patient doesn't have, are candidates for earlier screening, even at the first prenatal visit. Risk factors include history of previous pregnancy with gestational diabetes, polycystic ovarian syndrome (PCOS), history of macrosomia, high prepregnancy BMI, first-degree relative with diabetes mellitus type 2 and certain ethnic groups such as East Asian, Hispanic, and Pacific Islander. Syphilis and HIV testing are completed to decrease the risk of perinatal transmission. The blood type and antibody screen are used to identify mothers with blood antibodies that could cause hemolytic disease of the fetus. Mothers who are Rh negative will subsequently receive RhoGAM as well. Two maternal serum markers, hCG and pregnancy-associated plasma protein (PAPP-A … an early screen for Down syndrome), and one fetal marker (nuchal thickness) should be used between 11 and 14 weeks to evaluate for Down syndrome. This method offers a Down syndrome detection rate of approximately 85% with a 5% false-positive rate.

When Mrs Kline is 2 days postpartum The nurse would expect fundal height to be?
HELPFUL TIP:

Nuchal translucency looks at fluid collections in the posterior fetal neck. Increased translucency suggests increased fluid collection and is associated with abnormalities such as Down syndrome, Turner syndrome, and hemodynamic problems (cardiac abnormalities).

When Mrs Kline is 2 days postpartum The nurse would expect fundal height to be?
HELPFUL (AND IMPORTANT) TIP:

Since it does not screen for neural tube defects, first-trimester screening does not negate the need for a second trimester maternal serum triple or quad screen. The triple screen, which screens for chromosomal abnormalities (e.g., Down) and neural tube defects, includes levels for AFP (α-fetoprotein), HCG, and uE3 (unconjugated estriol). The quadruple screen adds an inhibin A. The triple or quadruple screen is done between 15 weeks 0 days and 19 weeks 6 days of gestation. The triple screen has a 70% sensitivity for neural tube defects and Down syndrome with a 5% false-positive rate. The quadruple screen has an 81% sensitivity and a 5% false-positive rate.

The patient has passed all of her screening tests with flying colors, and she returns at 36 weeks of gestation.

Question 15.1.8 What additional screening test(s) is/are obtained near 36 weeks of gestation?

A) Amniocentesis

B) 3-hour glucose tolerance test

C) Fetal fibronectin (FFN)

D) Group B streptococcus (GBS) culture or PCR

E) All of the above

Answer 15.1.8 The correct answer is "D." The CDC recommends maternal screening for GBS between 35 and 37 weeks of gestation. The only patients who should be excluded from screening are women who had bacteriuria with GBS during the current pregnancy or those who had an infant previously infected with GBS. These mothers are treated empirically with antibiotics in labor and do not need screening. Mothers whose culture or PCR status is unknown should receive antibiotic prophylaxis in labor only if one of the following exists: (1) intrapartum fever (>38°C), (2) preterm labor (<37 weeks of gestation), or (3) prolonged rupture of membranes (>18 hours).

Objectives: Did you learn to…

  • Describe contraception options?

  • Provide appropriate routine prenatal care?

  • Recognize nausea and vomiting of pregnancy and describe its appropriate management?

You are taking obstetric calls for your group over Labor Day (Get it? Bad joke?) weekend. Labor and delivery calls you about a 27-year-old G1P0 at 38 weeks of gestation who awoke this morning complaining of wetness. However, when she went to the bathroom she discovered significant vaginal bleeding that had soaked her bed. She denies any cramping or abdominal pain. She is on her way to the hospital—and so are you.

Question 15.2.1 You tell the nurses to call you with her vital signs and to initiate all of the following interventions immediately upon the patient's arrival EXCEPT:

A) Obtain IV access

B) Draw blood for type and screen

C) Perform a digital vaginal examination

D) Initiate fetal monitoring

E) Draw blood for complete blood count

Answer 15.2.1 The correct answer is "C." A small-to-moderate amount of bleeding is not unexpected during labor; however, the profuse bleeding described by the patient is an obstetric emergency. The first priorities are to obtain IV access and ensure that the mother is hemodynamically stable. Baseline laboratory evaluation will give some indication of the amount of blood loss and establish that blood is available for transfusion if necessary. Monitoring of the fetal heart rate (FHR) will evaluate fetal status. Also, an ultrasound should be done to evaluate for placenta previa. A digital vaginal examination should NOT be performed until the diagnosis of placenta previa has been excluded. A consultation with someone skilled in cesarean section should be obtained if the initial evaluation suggests that immediate fetal delivery is necessary.

When Mrs Kline is 2 days postpartum The nurse would expect fundal height to be?
HELPFUL TIP:

Classically, placenta previa presents as painless third-trimester bleeding, whereas placental abruption classically presents as painful third-trimester bleeding. Note that these are the "classic" presentations, not pathognomonic.

When the patient arrives at the hospital, she alters her recollection of events (never happens in our practice) to say that the fluid soaking the bed sheets was blood-tinged and pink in color and first occurred 2 hours ago. She continues to have vaginal leakage but denies any bright red bleeding or contractions. Ultrasound reveals a fundal placenta (not a placenta previa) without any evidence of abruption. Fetal heart tones are in the 140s and reactive. Sterile speculum examination reveals fluid, which is nitrazine and ferning positive (both evidence of ruptured membranes). Her GBS culture performed 3 weeks ago is negative. She is still not feeling contractions, and there are no contractions on the monitor.

Question 15.2.2 What is the most appropriate next step in the management of this patient?

A) Begin an induction of labor

B) Send her home after 4 hours of reassuring fetal monitoring

C) Treat her with IV penicillin for GBS prophylaxis

D) Repeat her GBS test to confirm GBS status

Answer 15.2.2 The correct answer is "A." A patient should not be sent home after rupture of membranes. She is at risk for intrauterine infection, and induction is indicated to reduce the risk of infection at term. Induction of labor, even with an unfavorable cervix, is not associated with an increase in cesarean or operative vaginal delivery, but it is associated with fewer maternal infections and fewer neonatal intensive care unit admissions. "C" and "D" are incorrect. Her GBS status was negative 3 weeks ago. Although GBS colonization can be transient, since the test was collected after 35 weeks it should be reliable. Even if her membranes were ruptured for more than 18 hours, she would not require treatment with antibiotics unless she developed a fever (remember she is GBS negative … review the criteria in answer 15.1.8). If a fever develops, think chorioamnionitis when choosing antibiotic therapy.

Sterile vaginal examination reveals a cervix that is 3 cm dilated, 2 cm long (effacement), soft, anterior, and vertex at a –1 station. The patient agrees to an induction of labor.

Question 15.2.3 The best induction method for this patient is to:

A) Insert intracervical laminaria

B) Begin IV oxytocin at 2 milliunits per minute

C) Insert intravaginal dinoprostone

D) Have her partner sit on her abdomen

E) Have her run a few laps around L&D (or walk around the hospital)

Answer 15.2.3 The correct answer is "B." The use of intravaginal and intracervical methods for cervical ripening could be cautiously considered in patients with premature rupture of membranes with an unfavorable cervix; but "C" is not the best choice—she is already 3cm dilated with a Bishop score of 6! Drugs for cervical ripening may include prostaglandins which act in the cervix to facilitate smooth muscle relaxation resulting in softening and dilatation of the cervix. They also may induce myometrial contraction. Currently the only prostaglandin labeled by the Food and Drug Administration (FDA) for such purpose are the prostaglandin E2 agents such as dinoprostone (Prepidil, Cervidil). Prostaglandin E1 agents such as misoprostol (Cytotec) inserted intravaginally are also commonly used for cervical ripening but do not carry a label by the FDA for this indication. Intracervical use of laminaria (an expanding pledget placed in the cervix) ("A") is generally reserved for pregnancy termination. "D" would not be comfortable and may cause undue harm to mom and the fetus, and is kind of weird. "E" is not the best choice. While walking may stimulate contractions, she already has ruptured membranes and a more aggressive approach is warranted.

Oxytocin should be carefully titrated via IV route for labor induction or augmentation. The general starting doses are 0.5 to 2 milliunits per minute, increased by 1 to 2 milliunits every 20 to 40 minutes, to a maximum dose rarely exceeding 30 to 40 milliunits per minute.

The patient is currently in labor (success!), and now her cervical examination is 6 cm dilation, 1 cm effaced, and –1 station. The amniotic fluid is still clear, having ruptured approximately 22 hours ago. She has had an epidural placed for analgesia. The FHR baseline has increased to 165 beats per minute with minimal variability. Contractions occur every 3 minutes. Maternal temperature is now 38.6°C, and her pulse is 110 bpm. The patient denies any complaints.

Question 15.2.4 Given the history of prolonged rupture of membranes and fever, which of the following is the most likely diagnosis?

A) Normal labor

B) Epidural fever

C) Nosocomial infection

D) Chorioamnionitis

Answer 15.2.4 The correct answer is "D." Chorioamnionitis is the most likely diagnosis and the diagnosis of most concern given the prolonged rupture of membranes (we gave you this answer above if you were reading …). Treatment should be initiated immediately. "A" is incorrect as fever is not a normal part of labor. "B" is possible, but not as likely and a dangerous assumption. There is an association between the use of epidural analgesia and a rise in maternal temperature. Etiologies proposed for this temperature increase include lack of pain-induced hyperventilation and decreased perspiration due to sympathetic blockade. "C" is extremely unlikely given her brief time in the hospital.

Question 15.2.5 What is the next step in the care of this patient?

A) Increase the oxytocin to hasten delivery

B) Remove the epidural

C) Initiate broad-spectrum antibiotics

D) Call your backup for cesarean section (or do it yourself)

E) Give acetaminophen 1,000 mg orally

Answer 15.2.5 The correct answer is "C." Initiation of antibiotics is associated with a decrease in both maternal and neonatal morbidity. Multiple organisms are isolated in more than 66% of cases; therefore, antibiotics should be broad. Approved regimens include ampicillin and gentamicin, ticarcillin/clavulanate, or piperacillin. There is no need to increase the oxytocin ("A"); she has already made good labor progress. There is also no need to proceed with a cesarean delivery ("D") unless there is another indication, such as nonreassuring FHR pattern. Although epidural anesthesia is associated with increased maternal temperature, it should only be removed if it is felt to be contributing to maternal pathology (e.g., meningitis, epidural abscess, or epidural bleed). Acetaminophen is fine, but it should not be used in lieu of antibiotics.

The patient's labor is progressing. Her cervix is 9 cm dilated, completely effaced, and station is +3. Her temperature is 39.0°C. The FHR pattern is shown in Figure 15-1.

When Mrs Kline is 2 days postpartum The nurse would expect fundal height to be?

FIGURE 15-1.

Question 15.2.6 What is the FHR interpretation?

A) Baseline 165 beats per minute, reactive

B) Baseline 165 beats per minute, with periods of bradycardia

C) Baseline 165 beats per minute, with late decelerations

D) Baseline 165 beats per minute, with variable decelerations

Answer 15.2.6 The correct answer is "C." The FHR pictured is described as Category II (see Table 15-2) with a baseline of about 165 beats per minute. There is moderate variability present. Following each contraction, there are late decelerations to the 110s. "A" is incorrect since "reactive" refers to a nonstress test, not fetal monitoring during labor (see Helpful Tip for more). "B" is incorrect as fetal bradycardia is defined as an FHR of less than 110 beats per minute for at least 10 minutes. "D" is incorrect. Variable decelerations vary with respect to timing, duration, and depth.

TABLE 15-2FETAL HEART TRACING CATEGORY DEFINITIONS

View Table||Download (.pdf)

TABLE 15-2 FETAL HEART TRACING CATEGORY DEFINITIONS

CATEGORY I Tracing (normal): FHR include ALL of the following:

  • Baseline FHR within normal range at 110–160 bpm

  • Moderate baseline FHR variability

  • Absent late or variable decelerations

  • Accelerations may be present or absent

CATEGORY II Tracing: Includes all FHR tracings that are not categorized as Category I or Category III. May represent an appreciable part of those encountered in clinical practice (atypical … May represent fetal hypoxia).

Any of the following:

  • Bradycardia with FHR <110 bpm WITH variability

  • Fetal tachycardia with FHR >160 bpm

  • Minimal baseline variability or absent baseline variability WITHOUT recurrent decelerations, or marked baseline variability

  • Absence of induced accelerations after fetal stimulation

  • Recurrent variable decelerations accompanied by minimal to moderate baseline variability

  • Prolonged deceleration >2 minutes but <10 minutes

  • Recurrent late decelerations with moderate baseline variability

  • Variable decelerations with other characteristics such as slow return to baseline, "overshoots," or "shoulders"

CATEGORY III Tracing (abnormal):

EITHER

  • Absent baseline FHR variability AND any of the following:

    • Recurrent late decelerations

    • Recurrent variable decelerations

    • Fetal bradycardia

OR

  • A sinusoidal pattern

When Mrs Kline is 2 days postpartum The nurse would expect fundal height to be?
HELPFUL TIP:

Nonstress tests are performed outside of the intrapartum period. A reactive nonstress test is defined as two accelerations of at least 15 beats above baseline and lasting at least 15 seconds within a 20-minute interval in gestations greater than 32 completed weeks. In gestations less than 32 weeks, reactive is defined as two accelerations at least 10 beats above baseline and lasting at least 10 seconds.

Question 15.2.7 What is the likely etiology of the fetal heart rate (FHR) tracing in Figure 15-1?

A) Head compression

B) Placental insufficiency

C) Cord compression

D) Any of the above is equally likely to cause the tracing

Answer 15.2.7 The correct answer is "B." Late decelerations are believed to be secondary to transient fetal hypoxia in response to decreased placental perfusion. Prompt evaluation and intervention is warranted. Early decelerations are generally reassuring and attributed to fetal head compression. Variable decelerations are the most common decelerations seen in labor and indicate cord compression. Variable decelerations can sometimes be relieved by maternal repositioning or amnioinfusion (infusing saline or Ringers Lactate to "reexpand" the uterus and relieve the pressure on the cord).

Question 15.2.8 What is the LEAST appropriate course of action now?

A) Administer maternal oxygen

C) Forceps or vacuum-assisted vaginal delivery

D) Consider cesarean delivery

E) Reposition mother (roll to one side or knee–chest)

Answer 15.2.8 The correct answer is "C." Increasing maternal oxygenation may improve fetal oxygenation. Oxytocin can decrease placental blood flow via uterine stimulation, and hence should be decreased or stopped if non-reassuring FHR changes are present. If there is evidence of maternal hypotension, maternal hydration may be indicated. Another option is position changes (left lateral) to improve placental perfusion. Maternal position can affect uterine blood flow and placental perfusion. The gravid uterus may compress the vena cava while supine. Because the patient is 9 cm dilated, forceps or vacuum-assisted vaginal delivery (also known as "operative vaginal delivery") is NOT indicated; dilation needs to be complete (10 cm) in order to use forceps or vacuum

The FHR changes resolve with your appropriate interventions. The patient progresses to complete dilation and delivers vaginally 1 hour later. Maternal antibiotics are discontinued following delivery, and the maternal temperature 2 hours after delivery is 37.0°C. On postpartum day 1, your patient complains of sore breasts from breastfeeding but no particular swelling of the breast (beyond what is expected postpartum), and her abdomen is sore "all over." She is having a moderate amount of lochia, and her temperature is 38.4°C.

Question 15.2.9 The most likely cause of the fever at this time is:

A) Endometritis

B) Mastitis

C) Deep vein thrombosis

D) Septic pelvic thrombophlebitis

Answer 15.2.9 The correct answer is "A." Whenever fever occurs in the immediate postpartum period, endometritis should be suspected. The presence of intra-amniotic infection (chorioamnionitis) increases the risk of postpartum endometritis to 13% (or 1 in 8 women, if you prefer fractions). Antibiotics are not routinely continued for chorioamnionitis after a vaginal delivery because the "source" of the infection (the placenta) has been removed. Mastitis ("B") is characterized by a swollen, firm, tender breast with systemic symptoms including fevers, chills, and flu-like symptoms. However, it was stipulated that the breasts were normal for our postpartum patient, and having bilateral mastitis is unlikely. Staphylococcus aureus is the typical pathogen in mastitis. Pregnancy and the postpartum period increase a woman's risk of thrombogenesis. However, DVT ("C") is not a likely source of the fever. Septic pelvic thrombophlebitis ("D") is a diagnosis of exclusion and is usually entertained when fever spikes continue despite treatment for endometritis.

You start appropriate antibiotics and the patient does well. The family, in gratitude, names the child after you (adding the suffix "-tops" to the child's name, like "Triceratops") and makes you the godparent necessitating gifts for the next 18 years on the child's birthday.

When Mrs Kline is 2 days postpartum The nurse would expect fundal height to be?
HELPFUL TIP:

Infants born to women with chorioamnionitis have a fourfold increase in neonatal mortality and a threefold increase in the incidence of respiratory distress, neonatal sepsis, and intraventricular hemorrhage.

Objectives: Did you learn to…

  • Triage and manage third-trimester bleeding?

  • Recognize and treat a patient with premature rupture of membranes?

  • Evaluate and manage intrapartum fever?

  • Interpret FHR patterns and begin initial management of abnormal patterns?

  • Appreciate the risk factors, evaluation, and management of postpartum infection?

You are seeing a 31-year-old G2P1 at 41 weeks of gestation by definite last menstrual period and 16-week ultrasound. She continues to note fetal movement and her examination is normal: BP 120/68 mm Hg, urine dipstick negative for protein and glucose, fundal height 42 cm, fetus is vertex, FHR 156 bpm. Her cervix is soft, anterior, 2 to 3 cm dilated, 50% effaced, and +1 station. She was induced with her first pregnancy, and this time she wants to have a "natural labor." You decide to calculate a Bishop score.

Question 15.3.1 The Bishop score helps to determine:

A) The health of the fetus

B) The need for cervical ripening agents for induction and helps to predict labor induction success

C) The maturity of the fetal lungs

D) The results of a Catholic intramural baseball game

Answer 15.3.1 The correct answer is "B." The Bishop score, which takes into account cervical dilation, effacement, consistency, and position, as well as fetal station, is a useful tool to determine if cervical ripening agents are needed for induction and to predict induction success. Calculators are readily available online.

The Bishop score is favorable at 9–10.

Question 15.3.2 Which of the following are the most appropriate recommendations at this point?

A) She should be induced at once; there is a high chance of fetal mortality after 41 weeks of gestation.

B) Since her antepartum course has been uncomplicated to date, it is safe for her to await spontaneous labor until 43 weeks of gestation.

C) She should undergo a nonstress test and ultrasound for amniotic fluid index.

D) She should plan for a cesarean section.

Answer 15.3.2 The correct answer is "C." By definition, a term gestation is one completed in 38 to 42 weeks. There is no significant increase in fetal mortality in an uncomplicated pregnancy at term. Virtually all reports suggest an increase in perinatal morbidity and mortality when pregnancy goes beyond 42 weeks of gestation. Antenatal surveillance of post-term pregnancies should be initiated at 41 weeks of gestation.

When Mrs Kline is 2 days postpartum The nurse would expect fundal height to be?
HELPFUL TIP:

Accurate determination of conception is important in reducing the false diagnosis of post-term pregnancy. The estimated date of delivery is most reliably and accurately determined early in pregnancy. Ultrasound may assist in determining dates, but has a standard of error that is dependent on the gestational age (see Table 15-3).

TABLE 15-3GUIDELINES FOR DATING BASED ON ULTRASONOGRAPHY

View Table||Download (.pdf)

TABLE 15-3 GUIDELINES FOR DATING BASED ON ULTRASONOGRAPHY

Gestational Age Range Based on LMPMethod of MeasurementDiscrepancy Between Ultrasound Dating and LMP Dating that Supports Re-dating

≤13 6/7 weeks

  • ≤8 6/7 weeks

  • 9 0/7 weeks to 13 6/7 weeks

CRL
  • More than 5 days

  • More than 7 days

14 0/7 weeks to 15 6/7 weeks BPD, HC, AC, FL More than 7 days
16 0/7 weeks to 21 6/7 weeks BPD, HC, AC, FL More than 10 days
22 0/7 weeks to 27 6/7 weeks BPD, HC, AC, FL More than 14 days
28 0/7 weeks and beyonda BPD, HC, AC, FL More than 21 days

Question 15.3.3 At 41 weeks she is really (really, really) tired of being pregnant and wants "a natural way" to induce contractions. Which of the following nonpharmacologic methods of inducing or augmenting labor is LEAST likely to be effective?

A) Stripping the amniotic membranes

B) Prolonged walking

C) Amniotomy

D) Nipple stimulation

Answer 15.3.3 The correct answer is "B." Stripping membranes appears to be effective in initiating spontaneous labor within 72 hours. Amniotomy may be used for labor induction, especially if the Bishop score is favorable. However, a Cochrane review from 2013 did not find strong evidence supporting the use of amniotomy alone or in combination with oxytocin for induction. Nipple stimulation causes release of oxytocin and may be utilized for labor induction, but its marginal benefit is only seen in patients with a favorable Bishop score. Walking does not result in labor induction or augmentation, but it's not harmful either.

When Mrs Kline is 2 days postpartum The nurse would expect fundal height to be?
HELPFUL TIP:

Sexual intercourse is sometimes recommended to induce labor. Studies are of low quality and use various endpoints … also, it is difficult to standardize the intervention (we could make a joke here but won't). One of the better quality studies (Tan et al., 2006) did find that coitus was associated with reduced need for labor induction at 41 weeks.

Question 15.3.4 If induction becomes necessary, which of the following pharmacologic interventions would be the best approach to your patient who has a cervix that is soft, anterior, 2 to 3 cm dilated, 50% effaced, and+1 station?

B) Intracervical PGE2 (dinoprostone)

C) Intravaginal PGE2 (dinoprostone)

D) Intravaginal PGE1 (misoprostol)

E) None of the above. All pharmacologic interventions are contraindicated

Answer 15.3.4 The correct answer is "A." This patient does not need further cervical ripening but is a candidate for induction of labor. If cervical ripening were needed, there are several available agents. Option "D," PGE1 (misoprostol, brand name Cytotec) can be administered intravaginally or orally (but note that the Food and Drug Administration [FDA] has not approved it for use in pregnancy). "B" and "C", PGE2 (dinoprostone, brand name Cervidil) is administered intravaginally. PGE2 gel (dinoprostone, brand name Prepidil) can be administered either intravaginally or intracervically. Because the cervix is favorable in this case, proceeding with oxytocin is the best option.

Your patient's husband is called up for active duty in Iraq (or Afghanistan … or Libya or … sadly, we are on the 4th edition of this book and have not needed to change this scenario) and is due to report in the next few days. She is now 41 2/7 weeks of gestation and desires induction so he can be with her for the delivery. You admit her to labor and delivery the following morning. The initial FHR monitoring before induction (also known as a nonstress test) is shown in Figure 15-2.

When Mrs Kline is 2 days postpartum The nurse would expect fundal height to be?

FIGURE 15-2.

Question 15.3.5 What is the correct interpretation?

A) Baseline 150 beats per minute; not reactive

B) Baseline 150 beats per minute; reactive

C) Baseline 180 beats per minute; decelerations to 150s; not reactive

D) Baseline 180 beats per minute; moderate variability; reactive

Answer 15.3.5 The correct answer is "B." The baseline is about 150 beats per minute. There are two accelerations greater than 15 beats and lasting longer than 15 seconds, which meets the criteria for a reactive nonstress test. There is one contraction and evidence of uterine irritability noted as well.

When Mrs Kline is 2 days postpartum The nurse would expect fundal height to be?
HELPFUL TIP:

When interpreting FHR tracings, variability is an important element that demonstrates fetal cardiac response to parasympathetic input. The small waveform fluctuations within the baseline heart rate tracing represent the FHR variability. After 28 weeks of gestation, variability should be present. It is categorized as absent (no amplitude, flat tracing), minimal (0–5 beat amplitude), moderate (6–25 beat amplitude), or marked (>25 beat amplitude). The absence of variability is associated with fetal decompensation or distress.

You perform amniotomy with return of particulate meconium-stained fluid. Her cervix is now 5 cm dilated, 80% effaced, with vertex at +1 station. You elect to continue monitoring progress.

Question 15.3.6 Which of the following choices of labor analgesia is MOST appropriate at this point?

A) Epidural analgesia

B) Local perineal anesthetic infiltration

C) Bilateral pudendal nerve block

D) All of the above are equally appropriate

Answer 15.3.6 The correct answer is "A." Epidural analgesia offers the most effective form of pain relief and generally may be utilized once the patient is determined to be in active labor. Various local anesthetic agents are available for local infiltration of the perineum and vagina to provide analgesia for episiotomy or laceration repair following delivery but not for labor. Bilateral pudendal nerve blocks are useful during the second stage of labor, as a supplement to epidural analgesia for anesthesia of the sacral nerves, or as an option for operative vaginal delivery anesthesia (e.g., forceps, vacuum). Opioid agonists and agonist–antagonists are also available and commonly employed. However, some reports suggest that the analgesic effect of opioids in labor is limited when using the lower doses that are generally regarded as safer for the fetus.

The nurse notices some changes on the fetal heart monitor. The current FHR is shown in Figure 15-3.

When Mrs Kline is 2 days postpartum The nurse would expect fundal height to be?

FIGURE 15-3.

Question 15.3.7 What is the correct interpretation of this FHR tracing?

A) Baseline 160 beats per minute; reactive

B) Baseline 160 beats per minutes; variable deceleration to the 90s

C) Baseline 160 beats per minute; late decelerations to the 90s

D) Baseline 160 beats per minute; early decelerations to the 90s

Answer 15.3.7 The correct answer is "B." A variable deceleration to the 90s occurs with the first contraction on this strip. Variable decelerations vary with respect to timing, duration, and depth—thus, the name "variable." They are not uniform. Variable decelerations represent changes in the FHR in response to cord compression. Please refer to the Table 15-2 with definitions and FHR tracing categories.

When Mrs Kline is 2 days postpartum The nurse would expect fundal height to be?
HELPFUL TIP:

A systematic review in 2013 showed no benefit in outcomes for continuous FHR monitoring compared to intermittent FHR monitoring. Unfortunately, there are more cesarean sections and operative vaginal deliveries when continuous FHR monitoring is used. However, much of FHR monitoring use is dictated by local practice patterns, expert consensus, and medicolegal concerns.

Question 15.3.8 Given the findings in Figure 15-3, which of the following should be performed next?

A) Check the patient's cervix

B) Place a fetal scalp electrode

C) Begin IV oxytocin infusion

D) Place an intrauterine pressure catheter and begin an amnioinfusion

Answer 15.3.8 The correct answer is "A." Variable decelerations are common in labor, and brief variable decelerations are benign. When variable decelerations become recurrent, progressively deeper, and longer lasting with delayed return to baseline, they are non-reassuring and may reflect fetal hypoxia. A pelvic examination should be performed to determine if the umbilical cord is prolapsed or if there has been rapid descent of the fetal head or rapid progression of labor. Oxytocin, "C," should not be considered in this patient since she is having adequate contractions. Replacement of the amniotic fluid ("D,'' amnioinfusion) with normal saline infused through a transcervical catheter has been reported to decrease both the frequency and severity of repetitive variable decelerations and can decrease rate of cesarean section. However, it would first be helpful to assess the cervical status. Of note, amnioinfusion is no longer recommended as a prophylactic intervention for moderate or severe meconium.

Labor progresses without incident. Your patient is now completely dilated and effaced, with fetal head at +3 station. She is comfortable with her epidural and able to push with good effort. The FHR tracing is reassuring. Contractions are every 3 minutes.

Question 15.3.9 Appropriate management at this point is:

A) Continue pushing

B) Vacuum-assisted delivery

C) Forceps-assisted delivery

D) Midline episiotomy

Answer 15.3.9 The correct answer is "A." At this point, labor is progressing and maternal–fetal status is reassuring. You should continue expectant management. No intervention is indicated.

When Mrs Kline is 2 days postpartum The nurse would expect fundal height to be?
HELPFUL TIP:

Episiotomies should not be performed routinely. Indications for episiotomy are typically related to non-reassuring fetal status and dystocia. There is no evidence that episiotomies reduce perineal trauma, postpartum dyspareunia, etc.

She pushes for 3 hours. She is now exhausted. The fetal head now separates the labia with contractions, and then recedes slightly. You consider offering assistance with delivery.

Question 15.3.10 In counseling your patient and her husband about the maternal risks of operative vaginal delivery, which of the following should you discuss?

A) Vaginal trauma

B) Shoulder dystocia

C) Fetal injury

D) Perineal and rectal trauma

E) All of the above

Answer 15.3.10 The correct answer is "E." Maternal risks of operative vaginal delivery include injury to the lower genital tract and rectal sphincter involvement in the case of a third- or fourth-degree laceration. In addition, fetal complications need to be discussed as well. Shoulder dystocia is more common with operative delivery than with a spontaneous vaginal delivery (see below for more information).

Question 15.3.11 Each of the following is a fetal risk of operative vaginal delivery EXCEPT:

A) Cephalohematoma

B) Skull fracture

C) Brachial plexus injury

D) Respiratory distress syndrome

E) Facial nerve palsy

Answer 15.3.11 The correct answer is "D." Respiratory distress syndrome is not increased by assisted delivery. Neonatal cephalohematoma, retinal hemorrhage, and jaundice (secondary to breakdown and reabsorption of the cephalohematoma) are more common with vacuum-assisted delivery than with forceps-assisted delivery. Skull fracture and facial nerve injury is more common with forceps-assisted delivery than with vacuum-assisted delivery. Shoulder dystocia with resultant brachial plexus injury is more common with vacuum-assisted delivery, prolonged time required for delivery, and increasing birth weight. Note that injury can occur before operative delivery as a result of abnormal labor forces (we don't think anyone has told the malpractice attorneys …).

Delivery of an 8-lb baby is accomplished without operative vaginal assistance. The mere presence of the vacuum on the table was enough to entice the uterus to perform one last massive contraction—assisted by the infant clawing its way out when it saw the vacuum coming. Following spontaneous delivery of the intact placenta 15 minutes later, you note a large gush of blood.

Question 15.3.12 Which of the following is the most likely source of the bleeding?

A) Uterine atony

B) Vaginal laceration

C) Cervical laceration

D) Retained placenta

Answer 15.3.12 The correct answer is "A." Postpartum hemorrhage is most commonly associated with uterine atony. Risk factors include prolonged labor, over-distended uterus (such as from 2 or 8 gestations [remember Octomom?]), very rapid labor, high parity, chorioamnionitis, retained placental tissue, poorly perfused myometrium, halogenated hydrocarbon anesthesia, and previous uterine atony. Maternal trauma to the genital tract ("B" and "C") may result in postpartum hemorrhage and should be routinely investigated, particularly following operative delivery. A retained placenta cotyledon is another common source for postpartum hemorrhage. The placenta should be inspected, and if there is any question of retained products of conception, the uterus should be manually explored.

Question 15.3.13 Which of the following should be undertaken next?

A) Obtain IV access and initiate hydration

B) Begin bimanual uterine compression

C) Inspect vagina and cervix for lacerations

D) Obtain blood for type and screen for possible blood transfusion

E) All of the above

Answer 15.3.13 The correct answer is "E." Postpartum hemorrhage is an obstetrical emergency and must be addressed immediately. The gravid uterus receives 500 mL of blood per minute, which can lead to massive hemorrhage if not addressed quickly. Additional personnel should be notified to help with obtaining IV access and blood draws, while you quickly try to identify the source of bleeding.

After thorough exploration of the vagina and uterus, you suspect uterine atony is the cause of bleeding. While continuing uterine massage, you think about your options.

Question 15.3.14 Which of the following is/are options in treating this patient's bleeding?

B) Methylergonovine (Methergine) IM

C) Carboprost tromethamine (Hemabate) IM

E) All of the above

Answer 15.3.14 The correct answer is "E." All of the drugs listed cause smooth muscle contraction of the uterus. Oxytocin can be given as a dilute IV solution or IM. It should never be administered as an undiluted IV bolus, due to the risk of hypotension and cardiac arrhythmia. Methergine (methylergonovine) is an ergot alkaloid and may be administered orally or intramuscularly (not intravenously). Caution should be used in women with hypertension, as Methergine can cause hypertension. Hemabate (carboprost tromethamine) is an F-2 prostaglandin analog that is administered IM or directly into the uterine myometrium. Caution should be used in women with asthma, as Hemabate can cause bronchoconstriction. Misoprostol is a prostaglandin E1 analog, that works well and can safely be administered to women with asthma or hypertension. Rectal or oral administration can be used, but rectal administration is preferred in a patient with potential hemodynamic instability. This can be a lifesaver especially in third world countries where other options may not exist.

She requires IV crystalloid and 4 units of packed red cells for symptomatic anemia following delivery. Both mother and infant do well and the patient and baby are discharged on postpartum day 2. You schedule a follow-up appointment in 2 days. You are concerned about Sheehan syndrome given the severe postpartum hemorrhage.

Question 15.3.15 All of the following are characteristic of Sheehan syndrome EXCEPT:

A) Failure in lactation

B) Amenorrhea

C) Desire to be a punk rocker

D) Decreased LH/follicle-stimulating hormone (FSH)

E) Adrenal cortical insufficiency

Answer 15.3.15 The correct answer is "C." Severe intrapartum or postpartum hemorrhage may result in pituitary necrosis due to hypovolemia and hypoperfusion. This leads to a global hypopituitarism known as Sheehan syndrome. Sheehan syndrome is characterized clinically by endocrine deficiency syndromes as a result of loss of anterior pituitary function. Initial symptoms may be vague (lethargy, anorexia, weight loss, difficulty with lactation), and the syndrome can go unrecognized. Later manifestations include failure of lactation, amenorrhea, breast atrophy, loss of pubic and axillary hair, adrenal cortical insufficiency, and hypothyroidism. Desire to be a punk rocker is "Sheena syndrome." If you don't get it, you missed the Ramones.

Objectives: Did you learn to…

  • Recognize the risks of prolonged pregnancy and identify appropriate timing of intervention?

  • Describe the indications and risks associated with induction of labor?

  • Interpret intrapartum FHR patterns and choose appropriate management options?

  • Evaluate analgesia options, contraindications, and risks during labor and delivery?

  • Recognize the indications for and management of operative vaginal and abdominal delivery?

  • Evaluate and manage postpartum hemorrhage?

When Mrs Kline is 2 days postpartum The nurse would expect fundal height to be?

While on call for your small community hospital, a nurse on the labor and delivery ward calls you about a patient with preterm contractions. You come in to see the patient and find a 33-year-old G1P0 at 26 weeks of gestation by in vitro fertilization. She is usually followed at an academic hospital 400 miles away. She looks worried and says, "I'm going to have twins. But not now!" She recalls her prenatal lab results were unremarkable. She recalls her ultrasound showed "two heart beats."

All of the following risks are increased with a multifetal gestation EXCEPT:

A) Preterm labor

B) D-isoimmunization (Rh isoimmunization)

C) Preterm rupture of membranes

D) Intrauterine growth restriction

E) Twin–twin transfusion syndrome

The correct answer is "B." The most significant complication of multiple gestations is preterm labor resulting in preterm delivery. Preterm rupture of membranes and intrauterine growth restriction also occur more frequently in multiple gestations than singleton pregnancies. The risk of all these complications is directly proportional to the number of fetuses. Twin–twin transfusion syndrome rarely occurs and is associated with mono-chorionic gestations. Multifetal gestations do not increase the risk of D-isoimmunization.

After a busy day in clinic, you get the traditional 5 PM call from labor and delivery. The nurse tells you that your partner's patient is presenting with concerns for preterm labor. She is a 24-year-old G1 at 26 weeks by LMP and first-trimester ultrasound. She is having contractions but no vaginal discharge. The contractions started about an hour ago and are 5 to 6 minutes apart. The nurse asks if you want to check fetal fibronectin (FFN).

Question 15.4.1 A NEGATIVE fetal fibronectin is associated with:

A) Fetal lung immaturity

B) Ruptured fetal membranes

C) A decreased risk of preterm birth

D) An increased risk of preterm birth

Answer 15.4.1 The correct answer is "C." FFN is a basement membrane protein produced by the fetal membranes. A negative test is useful in assessing the risk of preterm delivery during the following 2-week period. With a properly performed test in a symptomatic patient, up to 99.5% of patients with a negative FFN will not deliver in the subsequent 7 days. A positive test is not useful as the test has low positive predictive value. In performing FFN testing, the following criteria must be met: intact amniotic membranes, minimal cervical dilation (<3 cm), and sampling between 24 0/7 and 34 6/7 weeks. Recent sexual intercourse and the presence of vaginal discharge or bleeding may cause a false positive test. Collect the swab in the posterior vaginal fornix before cervical checks and transvaginal ultrasound as these can also cause the test to be falsely positive. FFN does not assess fetal lung maturity ("A"). Ruptured membranes ("B") would cause a positive FFN.

You are wondering if this patient is a good candidate for corticosteroid therapy.

Question 15.4.2 Regarding the risks and benefits of corticosteroid therapy for fetal lung maturation, which of the following is FALSE?

A) Corticosteroid therapy is recommended for all pregnant women between 24 and 34 weeks of gestation who are at risk of preterm delivery within 7 days

B) Corticosteroid therapy has been associated with an increased risk of neonatal infection

C) Antenatal corticosteroid therapy reduces the incidence of respiratory distress syndrome, intraventricular hemorrhage, and necrotizing enterocolitis

D) Corticosteroids accelerate the production of pulmonary surfactant in the fetal lungs

Answer 15.4.2 The correct answer is "B." There is no evidence that antenatal corticosteroid therapy increases the risk of neonatal infection. However, maternal infection is a relative contraindication to corticosteroid therapy. Corticosteroids are recommended for all pregnant women between 24 and 34 weeks of gestation who are at risk of preterm delivery within 7 days. Corticosteroids may be given after 34 weeks if there is documented fetal lung immaturity and delivery will likely occur before lung maturation.

Question 15.4.3 Which of the following FDA-approved tocolytics is regarded as first-line therapy to abort preterm labor?

B) Magnesium sulfate

C) Prostaglandin inhibitors

D) Ritodrine

E) None of the above

Answer 15.4.3 The correct answer is "E." The only FDA-approved agent for use as a tocolytic was ritodrine, a β-adrenergic receptor agonist, which was pulled from the market in the United States in 1993. There are no currently available FDA-approved tocolytics. Thus, other agents have been investigated. Terbutaline, magnesium sulfate, prostaglandin inhibitors, and calcium channel blockers have all been studied and may be utilized in select cases for tocolysis—but realize that these drugs are not approved by the FDA for this indication and that they carry substantial risks. Indomethacin has also been used successfully for tocolysis. Again, it is not approved for this indication.

When Mrs Kline is 2 days postpartum The nurse would expect fundal height to be?
HELPFUL TIP:

Tocolysis only prolongs labor by 48 hours at best, so the real goal should be to prolong the pregnancy until steroids have time to be effective. Contraindications to tocolysis include evidence of fetal distress, fetal anomalies, abruptio placentae, placenta previa with heavy bleeding, and severe maternal disease.

You examine the patient after collecting a FFN. Her cervix is 1 cm dilated at the external os and closed at the internal os, long, and posterior. Ultrasound shows that the infant is vertex. The monitor shows FHR baselines of 140 and 145 bpm. Contractions are irregular, occurring every 4 to 9 minutes. Urine dipstick shows a specific gravity of 1.030.

Question 15.4.4 The LEAST appropriate intervention in this 26 week gestation is:

A) Continuation of monitoring

B) Oral hydration

C) Obstetrical consult for cerclage placement

D) Test for group B streptococci, gonorrhea, and chlamydia

E) Administration of corticosteroids

Answer 15.4.4 The correct answer is "C." Cerclage (stitch to hold the cervix closed) is indicated for incompetent cervix, not preterm labor. A cerclage is typically placed in the first part of the second trimester after fetal viability has been established. It would not be used at 26 weeks. The patient should be evaluated carefully and the frequency of uterine contractions should be assessed during the initial management. Because dehydration may result in uterine irritability, rehydration may stabilize the uterus. There is no proven benefit of hydration in the patient who is euvolemic. Get cultures to prevent perinatal transmission even though treatment of positive cultures has not been established to aid in the prevention of preterm birth. Corticosteroids are discussed earlier.

When Mrs Kline is 2 days postpartum The nurse would expect fundal height to be?
HELPFUL (AND USELESS IF NOT AT LEAST INTERESTING) TIP:

Delivery rates in Israel go up around Jewish holiday fast days likely because of the relative dehydration.

Objectives: Did you learn to…

  • Recognize preterm labor and manage it appropriately?

  • Understand the indications, contraindications, risks, and benefits of tocolytics and corticosteroids?

A 33-year-old G1 at 35 2/7 weeks presents to labor and delivery. Her pregnancy has been complicated by preterm labor. Her GBS status is unknown. On admission, she is uncomfortable with regular contractions every 3 to 4 minutes. The FHR baseline is 135 with moderate variability. Her cervix is 6 cm dilated, completely effaced, with a bulging amniotic sac and fetus in vertex presentation.

Question 15.5.1 Which of the following is your best course of management?

A) Administer corticosteroids

B) Administer tocolytics

C) Initiate GBS prophylaxis

D) Discharge her to home until she is in active labor

E) A, B, and C

Answer 15.5.1 The correct answer is "C." GBS prophylaxis is indicated for preterm delivery if GBS status is unknown. Appropriate antibiotics include penicillin or ampicillin or an alternative intravenous agent if the patient has a penicillin allergy. This patient is outside the window (24–34 weeks) for corticosteroid administration. Given the gestational age (>34 weeks), advanced cervical dilation, and high likelihood of imminent delivery, tocolysis should be avoided.

When Mrs Kline is 2 days postpartum The nurse would expect fundal height to be?
HELPFUL TIP:

Indications for GBS prophylaxis: (1) GBS screen positive during current pregnancy; (2) GBS bacteriuria anytime during current pregnancy; (3) prior history of giving birth to a neonate with GBS disease; (4) unknown GBS status plus fever or preterm labor, or prolonged rupture of membranes. No GBS prophylaxis is indicated in women who are culture negative with fever (but think about chorioamnionitis!), preterm labor, or prolonged rupture of membranes.

When Mrs Kline is 2 days postpartum The nurse would expect fundal height to be?
HELPFUL TIP:

The risk for recurrent premature delivery in subsequent pregnancies is about 15%. 17α-Hydroxyprogesterone caproate (brand name Makena) may reduce the incidence of preterm birth when started at 16 to 20 weeks and can be offered to a woman with a singleton gestation and a history of a prior spontaneous preterm singleton birth. Recent data suggests it may be less effective than once thought (Lancet. 2016; Feb 23). If prior preterm birth was secondary to an incompetent or shortened cervix, then a cerclage should also be considered (ACOG. Obstet Gynecol. 2012;120(4):964–973).

Objective: Did you learn to…

  • Describe indications for GBS prophylaxis?

When Mrs Kline is 2 days postpartum The nurse would expect fundal height to be?

Risk factors for preterm birth include each of the following EXCEPT:

A) Multiple gestation pregnancy

B) Maternal bacteriuria

C) Maternal history of preterm contractions with term birth

D) Maternal smoking

E) Maternal hypertension

The correct answer is "C." While preterm contractions are concerning, a maternal history of preterm contractions with term birth does not increase the risk of preterm delivery in a subsequent pregnancy. Multiple gestations, history of preterm birth, smoking, cocaine use, asymptomatic bacteriuria, and hypertension are all risk factors for preterm birth.

A 37-year-old G3 P0111 (full term, preterm, abortions/miscarriage for any reason, living children) presents for routine obstetric care at 10 weeks of gestation. Her second pregnancy was complicated by preeclampsia with severe features at 35 weeks requiring induction of labor and magnesium sulfate therapy. Her past medical history is uncomplicated. Her blood pressure is 132/86 mm Hg, urine protein is negative, and physical examination is unremarkable. Uterine size is consistent with dates and fetal heart tones are auscultated. The patient wonders if she will need to deliver early and need magnesium again.

Question 15.6.1 You counsel her that her risk of recurrent preeclampsia with severe features is in the range of:

A) <5%

B) 20% to 30%

C) 50% to 60%

D) >90%

Answer 15.6.1 The correct answer is "B." The risk of recurrence of preeclampsia is affected by both gestational age at diagnosis and the severity of preeclampsia. Preeclampsia at an early gestational age or preeclampsia with severe features increases the risk of recurrence. The overall recurrence risks are estimated at less than 10% for mild preeclampsia and greater than 20% for preeclampsia with severe features. Risk factors for preeclampsia include young maternal age, advanced maternal age, diabetes, and chronic hypertension, among many others.

The patient would like to do anything reasonable to prevent preeclampsia again.

Question 15.6.2 In addition to obtaining baseline laboratory evaluation (CBC, AST, ALT, creatinine, 24-hour urine for protein) to aid in early diagnosis, you recommend the following therapy:

A) Low-sodium diet

B) Diuretic for hypertension and edema

D) Subcutaneous heparin therapy at prophylactic doses

E) None of the above

Answer 15.6.2 The correct answer is "C." ACOG's Task Force on Hypertension in Pregnancy from November 2013 recommends the use of 81 mg aspirin daily in patients who have a history of early-onset preeclampsia with preterm delivery or in patients with a history of preeclampsia in more than one prior pregnancy. USPSTF recommends the use of low-dose aspirin as a preventive medication after 12 weeks of gestation in women who are at high risk for preeclampsia.

Because of her age (37 years), the patient also inquires about her risk for delivering a baby with Down syndrome.

Question 15.6.3 What is her estimated risk of a Down syndrome baby with this pregnancy?

A) 1/9,000

B) 1/1,200

C) 1/150

D) 1/12

Answer 15.6.3 The correct answer is "C." The risk of Down syndrome begins to rise rapidly at age 35, with an estimated risk of 1/250 at age 35, 1/150 at age 37, and 1/70 at age 40.

After hearing her age-related risk for Down syndrome, the patient asks about what tests she should have to screen for Down syndrome.

Question 15.6.4 You counsel her regarding various tests and offer:

A) First-trimester triple screening (PAPP-A, hCG, and nuchal lucency)/integrated screening

B) Chorionic villus sampling (CVS)

C) Amniocentesis

D) Second-trimester quadruple screening (hCG, maternal serum alpha fetal protein [MSAFP], estriol, inhibin A)

E) Cell-free fetal DNA testing from maternal peripheral blood

F) All of the above

Answer 15.6.4 The correct answer is "F." All patients should be offered first-trimester screening, and if declined, they should be offered second-trimester quadruple screening. Because of her advanced age (>35 years at the time of delivery), the patient should be offered CVS and amniocentesis as well. Patients should understand that the first- and second-trimester screening tests are just that—screening methods. However, both CVS and amniocentesis can diagnose Down syndrome, in addition to other chromosomal abnormalities. CVS is typically completed between 10 and 13 weeks of gestation, whereas amniocentesis is performed after 15 weeks. Both are invasive procedures, which carry risks, including pregnancy loss, rupture of membranes, and fetal injury. Cell-free fetal DNA testing is a newer and noninvasive screening test that can be done after 10 weeks of gestation in patients at high risk for aneuploidy. If abnormal, it should be followed up with a diagnostic test, such as an amniocentesis.

The patient has a negative first-trimester screening, which decreases her Down syndrome risk to 1/800. She declines an amniocentesis. You draw her MSAFP only at 17 weeks, because, as you learned above, the first-trimester screening does not evaluate for neural tube defects. The AFP is normal. She subsequently undergoes diabetes screening with a 1-hour post-50-g glucose test, which shows a glucose level of 170 mg/dL.

Question 15.6.5 The next step is:

A) Order a 3-hour glucose tolerance test

B) Set up diabetes teaching and a consult with a nutritionist/dietician

E) All of the above

Answer 15.6.5 The correct answer is "A." The 1-hour 50-g glucose load test is a screening test for gestational diabetes. Since hers is abnormal (≥140 mg/dL), the next step is to perform a diagnostic test, which is the fasting 3-hour glucose tolerance test, utilizing a 100-g glucose load.

When Mrs Kline is 2 days postpartum The nurse would expect fundal height to be?
HELPFUL TIP:

The cutoff for the 1-hour 50-g glucose load (Glucola) test is either ≥130 or ≥140 mg/dL, and either is currently acceptable. The International Association of Diabetes in Pregnancy Study Group has proposed new screening guidelines for gestational diabetes. These would be more aggressive, including universal screening (with fasting glucose or A1C) at the initial visit and a glucose level 2 hours after a 75-g glucose load in the second trimester that would be used for screening and diagnosis.

The patient completes her glucose tolerance test with values of:

  • Fasting: 92 mg/dL

  • 1 hour: 194 mg/dL

  • 2 hours: 169 mg/dL

  • 3 hours: 148 mg/dL

Question 15.6.6 The next step in management is:

A) Continue routine prenatal care

B) Set up diabetes teaching and a consult with a nutritionist/dietician

Answer 15.6.6 The correct answer is "B." The recommended upper limit of normal serum glucose levels for the 3-hour glucose tolerance test are:

  • Fasting: 95 mg/dL

  • 1 hour: 180 mg/dL

  • 2 hours: 155 mg/dL

  • 3 hours: 140 mg/dL

If a patient has two or more glucose levels that are above these, she is diagnosed with gestational diabetes (GDM). Your patient fails on three of four of her results. The recommendation is to initiate dietary modifications including carbohydrate restriction with frequent blood sugar monitoring. If the target glucoses cannot be met with dietary changes alone, medical therapy should be started. Although insulin has been the standard therapy, glyburide or metformin are also equally appropriate for first-line therapy. Insulin is better than metformin, which is better than glyburide (BMJ. 2015; 350:h202).

The patient is seen for her 32-week visit and her fundal height is only 28 cm (which surprises you, given her diagnosis of gestational diabetes … the babies of diabetic mothers tend to be large!). She has been compliant with the dietary changes and her blood sugars are usually 80s fasting and 120s 2-hour postprandial. She has gained 20 lb so far in the pregnancy. You send her for an ultrasound, which reveals an infant measuring only 28 3/7 weeks of gestation, weighing 1,168 g (<10th percentile). Amniotic fluid volume and umbilical artery Dopplers are normal.

Question 15.6.7 Appropriate follow-up includes:

A) Changing the estimated due date

B) Scheduling an induction

C) Repeating the ultrasound for growth in 1 week

D) Repeating the ultrasound for growth in 4 weeks

Answer 15.6.7 The correct answer is "D." Current ultrasound techniques are not sensitive enough to assess growth at weekly intervals ("C"), and therefore waiting 4 weeks would give a better assessment of growth rate. This infant demonstrates intrauterine growth restriction. The patient had a first-trimester ultrasound with her first-trimester screening, which establishes her due date. It is inappropriate to change her due date based on a 32-week ultrasound ("A"). Initiating an induction would be inappropriate without further investigation, given the early gestational age ("B"). You could do an ultrasound at 1 to 2 weeks to assess amniotic fluid and umbilical Doppler but not fetal size. Antenatal surveillance is indicated at this time with bi-weekly nonstress testing.

The patient continues with bi-weekly nonstress tests and her ultrasound at 35 weeks reveals appropriate interval growth, but remains growth-restricted at a weight of 1,846 g (<10th percentile). Today her blood pressure is 146/88 mmHg and urine protein on dipstick is +1. Ugh … she can't seem to catch a break.

Question 15.6.8 At this time, appropriate intervention includes:

A) Administering corticosteroids

B) Obtaining a 24-hour urine for protein

C) Following up with a routine appointment in 1 week

Answer 15.6.8 The correct answer is "B." Given the patient's elevated blood pressure and 1+ protein on dipstick, you need to be concerned about recurrent preeclampsia. Urinary excretion of protein may be transient, and a 24-hour urine protein level is a more accurate reflection of proteinuria and the preferred method to diagnose preeclampsia. Alternatively, a protein/creatinine ratio of at least 0.3 mg/dL can also be used for preeclampsia diagnosis. "A" is incorrect. The patient is at 35 weeks of gestation (although measuring smaller), which is beyond the recommended gestation at which corticosteroids are administered (24–34 weeks). This patient must be followed up in a couple of days so that you don't miss the diagnosis of preeclampsia. Although anti-hypertensives such as labetalol can be utilized in pregnancy, they are not routinely initiated for mild elevations in blood pressure at later gestations. We hope you avoided "E," as ACE inhibitors are contraindicated in pregnancy.

Serial blood pressure measurements in the clinic reveal no blood pressures greater than 146/88. Her nonstress test is reactive. The patient is sent home to collect her 24-hour urine and returns in 2 days. Her blood pressure is now 148/90 mm Hg and she has trace protein on urine dipstick. The 24-hour urine returns at 180 mg (her baseline 24-hour urine protein at the beginning of the pregnancy was 116 mg). She denies any headache, visual changes, nausea, or abdominal pain.

Question 15.6.9 Your diagnosis is:

A) Gestational hypertension

B) Preeclampsia

C) Preeclampsia with severe features

D) Acute renal failure

Answer 15.6.9 The correct answer is "A." She now has two blood pressure readings greater than 140/90 mmHg and more than 6-hours apart, which satisfy the criteria for hypertension. Given that this elevation in blood pressure started after 20 weeks of gestation, it is likely pregnancy related. She does not have protein >300 mg in a 24-hour urine collection, so she does not meet that diagnostic criterion for preeclampsia and does not have symptoms that would qualify her for a diagnosis of preeclampsia with severe features. Note that proteinuria is no longer required to make the diagnosis of preeclampsia. Another abnormality such as elevated LFTs, thrombocytopenia, renal insufficiency, CHF, or CNS symptoms can make the diagnosis of preeclampsia even in the absence of proteinuria. The diagnostic criteria for preeclampsia are outlined in Table 15-4.

TABLE 15-4DIAGNOSTIC CRITERIA FOR PREECLAMPSIA

View Table||Download (.pdf)

TABLE 15-4 DIAGNOSTIC CRITERIA FOR PREECLAMPSIA

2013 Diagnostic Criteria for Preeclampsia

Blood pressure:

  • ≥140 mm Hg systolic or >90 mm Hg diastolic on TWO occasions at least 4 hours apart AFTER 20 weeks of gestation in a woman with history of previously normal blood pressure

  • ≥160 mm Hg systolic or ≥110 mm Hg diastolic; hypertension can be confirmed within a short interval (minutes) to facilitate timely antihypertensive therapy

AND

  • Proteinuria:

    • ≥300 mg per 24-hour urine collection (or this amount extrapolated form a timed collection)

      OR

    • Urine protein:creatinine ratio greater than or equal to 0.3 with each measured in mg/dL

      OR

    • Dipstick reading of 1+ (used only if other quantitative methods aren't available)

OR

In the absence of proteinuria, new-onset hypertension with the onset of any of the following (laboratory values further defined under "Diagnosis of Preeclampsia with Severe Features below):

Thrombocytopenia

  • Renal insufficiency

  • Impaired liver function

  • Pulmonary edema

  • Cerebral or visual symptoms

Diagnose "Preeclampsia with Severe Features" when there is the presence of any of the following:

  • Blood pressure >160 mm Hg systolic or ≥110 mm Hg diastolic on two occasions at least 4 hours apart while the patient is on bed rest (unless antihypertensive therapy is initiated before this time)

  • Platelet count of less than 100,000/μL (thrombocytopenia)

  • Serum creatinine concentrations greater than 1.1 mg/dL or a doubling of the serum creatinine concentration in the absence of other renal disease (renal insufficiency)

  • Elevated blood concentrations of liver transaminases to 2× normal concentration, severe persistent right upper quadrant or epigastric pain unresponsive to medication and not accounted for by alternative diagnoses or both (impaired liver function)

  • Pulmonary edema

  • New-onset cerebral or visual symptoms

She returns to the office for her appointment 5 days later at 36 weeks of gestation after having felt well over the weekend. However, today she developed a headache. Her blood pressure is 166/112 and her urine dipstick reveals 3+ protein.

Question 15.6.10 Your next step is to:

A) Start oral labetalol and see the patient back in 2 days for a blood pressure check

B) Repeat the 24-hour urine for protein

C) Admit to labor and delivery for blood work and monitoring, with plans to move toward delivery

D) Immediate Cesarean section

Answer 15.6.10 The correct answer is "C." The clinical picture is now developing into preeclampsia with severe features (headache, systolic BP >160, diastolic BP >110, and 3+ proteinuria … see Table 15-4). The patient needs to be admitted for further monitoring of blood pressure and symptoms. In addition, blood work should be obtained including a CBC and liver and renal studies. Starting oral labetalol would treat the patient's hypertension, but this step alone is not prudent for a patient who appears to have preeclampsia with severe features. The 24-hour urine protein collection may be helpful in meeting the technical criteria to diagnose preeclampsia and can be done during admission; however, the results would not change the immediate management of the patient. If a faster evaluation is needed, the protein/creatinine ratio could be done in this case. However, as noted above, preeclampsia with severe features can also be diagnosed without the presence of proteinuria in the presence of certain clinical and/or laboratory findings (see Table 15-4).

When Mrs Kline is 2 days postpartum The nurse would expect fundal height to be?
HELPFUL TIP:

In a preeclamptic patient, blood pressure can be controlled with labetalol, hydralazine, or oral nifedipine. Avoid nitroprusside. The downside of BP control is that it reduces placental flow. Also, treating the BP has no effect on the course of preeclampsia. Therefore, treat BP only if >160/100 mm Hg or the patient is having end-organ symptoms. Further information on acute and emergent management of severe hypertension can be found in Am Obstet Gyencol. 2015;(623):1–5.

You admit her to the hospital. Repeat blood pressure is 164/98 mmHg and urine dipstick shows 3+ protein. Her cervix is soft, 2 cm dilated, 50% effaced, with fetus vertex at –2 station.

Question 15.6.11 What is the most appropriate intervention at this point?

A) Begin induction of labor

B) Start magnesium sulfate

C) Prepare for a cesarean delivery in case it is needed

D) All of the above

Answer 15.6.11 The correct answer is "D." All of these options are important to consider at this time. Induction with oxytocin and treatment of preeclampsia with magnesium are appropriate at this point. Obstetrical backup should be involved earlier rather than later unless you possess the skill to do the cesarean section yourself.

When Mrs Kline is 2 days postpartum The nurse would expect fundal height to be?
HELPFUL TIP:

Delivery of the baby (baby and placenta, really) is the ultimate treatment for preeclampsia and should be initiated as soon as feasible when the mother's condition demands it.

You begin magnesium sulfate for the preeclampsia and oxytocin for induction. The induction proceeds without incident, and she delivers a viable male infant.

Question 15.6.12 How long will you continue the magnesium sulfate?

A) Until delivery of the placenta

B) For 12 hours after delivery

C) For 24 hours after delivery

D) Until the urine protein dipstick is negative

E) Until discharge

Answer 15.6.12 The correct answer is "C." Treatment should be continued for 24 hours following delivery.

When Mrs Kline is 2 days postpartum The nurse would expect fundal height to be?
HELPFUL TIP:

Monitor deep tendon reflexes, level of consciousness, and urine output for all patients on magnesium. Turn off the magnesium infusion if signs of toxicity emerge (e.g., decreased mental status, hyporeflexia) or if the patient is at risk for impending toxicity (e.g., from renal failure).

When Mrs Kline is 2 days postpartum The nurse would expect fundal height to be?
ANOTHER HELPFUL TIP:

Remember all those "mag checks" you did every 2 hours as a medical student? Well, for patients with preeclampsia without severe features, ACOG no longer recommends magnesium sulfate as a routine intervention (again, look at Table 15-4). Not a total waste of time, right? "Mag checks" did improve your rapport with your patients … maybe. Remember that the purpose of magnesium sulfate infusion is to reduce the risk of seizures (i.e., the progression to eclampsia). The NNT for mild preeclampsia is 100.

When Mrs Kline is 2 days postpartum The nurse would expect fundal height to be?
YET ONE MORE HELPFUL TIP:

According to ACOG guidelines, bed rest should not be prescribed for patients with gestational hypertension or preeclampsia without severe features.

Objectives: Did you learn to…

  • Screen for, diagnose, and manage gestational diabetes?

  • Evaluate, diagnose, and manage hypertension in pregnancy?

  • Define and manage preeclampsia?

  • Identify intrauterine growth restriction?

Now, you have to rush from L&D back to clinic (this never happens, right?) where you meet a 31-year-old woman for preconception counseling. She has a history of hypertension and a heart murmur. She has dyspnea when climbing stairs but performs normal activities of daily living with minimal difficulty. She takes lisinopril 10 mg daily for her blood pressure. On physical examination, heart rate is 82 bpm and blood pressure is 138/90 mm Hg. Height is 5′ 6″ and weight is 160 lb. She appears well.

Question 15.7.1 Regarding the management of her chronic hypertension during pregnancy, which is the most appropriate next step?

A) Discontinue lisinopril and begin methyldopa, labetolol, or nifedipine and recheck blood pressure in 2 weeks

B) Increase lisinopril to 20 mg daily and recheck blood pressure in 2 weeks

C) Make no changes at this time

D) Discontinue lisinopril; recheck blood pressure in 2 weeks

E) Either A or D is correct

Answer 15.7.1 The correct answer is "E." ACE inhibitors are contraindicated in pregnancy; therefore, a woman contemplating pregnancy should discontinue the medication or replace it with a safer alternative. In fact, for a woman capable of conceiving, ACE inhibitor use is discouraged unless no better alternative exists and the patient is using a reliable form of birth control. Women with mild chronic hypertension (systolic blood pressure 140–160 mm Hg) have a low risk for cardiovascular complications during pregnancy and can be managed with nonpharmacologic therapy as long as asymptomatic. In 2 weeks, when she has her return visit, if she is hypertensive (>140–160/>100 mm Hg), select a medication regarded as safe during pregnancy; methyldopa, nifedipine, or labetalol would be preferred. You could also start one of these at the current visit while discontinuing the ACE inhibitor. Note that the ACOG recommendation is to treat chronic hypertension in pregnancy only if the systolic BP is >160/105 mm Hg. If the patient has hypertension secondary to preeclampsia, ACOG would recommend treating only if the blood pressure is greater than >160/110 mm Hg or if there are symptoms. These recommendations are controversial and many would initiate blood pressure control in both groups at a lower BP. Treating hypertension reduces the incidence of "severe" hypertension, but does not change outcomes (N Engl J Med. 2015;372:407–417). See also Obstet Gynecol. 2013;122:1122 if you are interested in this topic.

Question 15.7.2 Which of the following statements about cardiovascular physiology in pregnancy is INCORRECT?

A) Blood volume and cardiac output increase by approximately 50% during pregnancy

B) Heart rate increases by 10 to 20 beats per minute, peaking in the third trimester

C) Systemic arterial pressure increases during the first trimester, reaches a peak in mid-pregnancy, and remains at that level until labor and delivery

D) Left ventricular ejection fraction remains constant or increases slightly throughout pregnancy

E) A temporary rise in venous return immediately following delivery may lead to a substantial rise in left ventricular filling pressure and cardiac decompensation in women with certain types of heart disease

Answer 15.7.2 The correct answer (and what does not happen in pregnancy) is "C." Systemic arterial pressure decreases during the first trimester, reaches a nadir during the second trimester, and returns to prepregnancy levels in the third trimester. The other statements are true. "E" deserves special mention. Immediately following delivery, relief of caval compression may cause a rise in venous return leading to clinical deterioration in some women with heart disease.

Objectives: Did you learn to…

  • Manage hypertension during preconception counseling?

  • Recognize normal cardiac physiological changes associated with pregnancy?

You are seeing a 28-year-old female who's LMP was approximately 7 days ago. She is complaining of vaginal bleeding and states that she is going through one pad every 20 minutes. Needless to say she is concerned. She is not using anything for contraception. Her vital signs and examination are unremarkable except for blood at the os.

Question 15.8.1 After ruling out pregnancy and assuring that her hemoglobin is stable, what is your next step?

A) Fresh frozen plasma

B) Cyclic medroxyprogesterone 10 mg a day

C) DDAVP to maximize platelet function

D) Observation: nonintervention is the best policy if the hemoglobin is normal

Answer 15.8.1 The correct answer is "B." This patient has abnormal uterine bleeding, which is a broad term that includes different functional and structural causes of uterine bleeding. Dysfunctional uterine bleeding (DUB) secondary to anovulation is a very common cause of abnormal uterine bleeding. One popular regimen for DUB is medroxyprogesterone 10 mg/day for 10 days. This should be followed by a withdrawal bleed. This regimen can be repeated for the first 10 days of the next 3 months. Other options include starting a monophasic OCP (three pills twice a day for 1 day, two pills twice a day for 1 day, one pill twice a day for 1 day, and then finish out the pack). Conjugated estrogens are a third option. Tranexamic acid (an antifibrinolytic) 1.3 g orally TID for 5 days is an additional alternative for patients with contraindications to hormonal therapy.

Question 15.8.2 The most common side effect of medroxyprogesterone at the above dose includes:

A) Bone marrow suppression

B) Thromboembolic disease

C) Depression

D) Sore breasts

Answer 15.8.2 The correct answer is "C." Many women will become depressed when taking this dose of medroxyprogesterone. They should be made aware of this ahead of time. The major side effect of the OCP regimen for abnormal uterine bleeding is nausea and vomiting. Consider giving the OCP regimen with an antiemetic. "B" is of note. Progesterones alone do not increase clot risk. They do inhibit thrombolysis, however.

When Mrs Kline is 2 days postpartum The nurse would expect fundal height to be?
HELPFUL TIP:

Another long-term option for patients with chronic abnormal uterine bleeding, after workup has been completed, is Mirena IUD.

Question 15.8.3 Causes of this patient's bleeding could include all of the following EXCEPT:

A) Hypothyroidism

B) Von Willebrand disease

C) Prolactinoma

D) Parathyroid disease

E) Uterine cancer

Answer 15.8.3 The correct answer is "D." All of the remaining can cause abnormal bleeding. Obviously, uterine cancer would be unlikely in a 28-year-old. Other causes of bleeding to consider are cervical polyps, endometrial polyps, adenomyosis, fibroids, and endocrine problems such as PCOS and hypo/hyperthyroidism, etc.

Objectives: Did you learn to…

  • Evaluate a patient with abnormal uterine bleeding?

  • Treat a patient with abnormal uterine bleeding?

A 37-year-old G3 P1112 (full term, preterm, abortions/miscarriage from any cause, living children) presents for an annual gynecologic examination and has questions regarding contraception. She is wondering if she is a candidate for an IUD.

Question 15.9.1 All of the following are contraindications to IUD placement EXCEPT:

A) Pregnancy

B) Acute pelvic infection

C) Undiagnosed vaginal bleeding

D) History of chlamydia infection

Answer 15.9.1 The correct answer is "D." Although active cervicitis and acute pelvic inflammatory disease (PID) are contraindications to placement of an IUD, a prior chlamydia infection does not exclude a patient from obtaining an IUD. Undiagnosed vaginal bleeding can be a sign of either endometritis or structural abnormalities of the uterine cavity, which must be addressed prior to considering an IUD. If you chose "A," some goons from the Board are coming to relieve you of your certification.

Your patient has none of these contraindications, and she would like to proceed with the IUD. Her friends have told her that all IUDs cause abortions, and she is unsettled by this idea.

Question 15.9.2 The mechanism of action of IUDs (copper or progesterone) is:

A) Abortifacient

B) Causes a sterile inflammatory reaction to a foreign body

C) Impairs sperm transport from the cervix to the fallopian tube

D) B and C

E) All of the above

Answer 15.9.2 The correct answer is "D." Studies detecting levels of hCG reveal that this hormone is not present in IUD users during the luteal phase. Thus, the IUD is NOT an abortifacient. Studies suggest that the mechanism of action of IUDs includes interference with sperm transport from the cervix to the fallopian tube, inhibition of sperm survival, and endometrial inflammatory changes that inhibit implantation.

When Mrs Kline is 2 days postpartum The nurse would expect fundal height to be?
HELPFUL TIP:

The progesterone-releasing Mirena IUD is approved for 5 years of contraception versus 10 years for the copper IUD. Skyla is approved for only 3 years.

After all of that counseling (that you can't bill for), her husband undergoes a vasectomy. During a routine appointment 2 years later, she complains of worsening menorrhagia. She denies intermenstrual spotting. She has not noticed any lightheadedness or dizziness, but does complain of generalized fatigue. On physical examination, you find a normal sized thyroid and an enlarged, irregular uterus measuring 10 to 12 weeks in size. There are no distinct adnexal masses, but this is somewhat difficult to discern due to the irregular uterus.

Question 15.9.3 Your initial workup in this 39-year-old female should include all of the following EXCEPT:

A) CBC

B) TSH

C) CA-125

D) Pelvic ultrasound

E) Pregnancy test

Answer 15.9.3 The correct answer is "C." A CBC will provide information regarding the hematocrit (and therefore the level of anemia) and platelet count—important information for someone with heavy menstrual bleeding. TSH will screen for hypothyroidism, which is a common cause of menorrhagia in a 39-year-old female. A pelvic ultrasound will aid in the evaluation of the mass palpated on examination and characterize the location and size of any uterine fibroids that may contribute to the bleeding; it will also evaluate for any adnexal masses. A pregnancy test is always a good idea to rule out possible pregnancy, ectopic pregnancy, and miscarriage as an underlying etiology. CA-125 is a nonspecific tumor marker that has a high false-positive rate in premenopausal women. Thus, CA-125 is not recommended as a screening test for ovarian cancer. It can be used as an adjunct to pelvic ultrasound when a complex adnexal mass is identified.

The patient returns in 2 weeks following her ultrasound. Her hematocrit was 31% (indices are consistent with iron-deficiency anemia), platelets 195,000, and TSH 2.8 mU/L (normal). The ultrasound reveals a uterus measuring 12 × 8 × 6 cm, with multiple small intramural fibroids measuring less than 2 cm in diameter. There is one subserosal, pedunculated fibroid measuring 3.5 cm at the fundus.

Question 15.9.4 What is the most appropriate initial management?

A) Expectant management and reassurance

B) Nonsteroidal anti-inflammatory drugs (NSAIDs)

C) Gonadotropin releasing hormone (GnRH) agonists

D) Blood transfusion

E) Hysterectomy

Answer 15.9.4 The correct answer is "B." Most fibroids are asymptomatic, although the most common symptom associated with leiomyomata (fibroids) is abnormal bleeding. This patient's fibroids are not impinging on the uterine cavity but may be contributing to the patient's menorrhagia. Initial treatment of menorrhagia may include NSAIDs that inhibit prostaglandin synthesis. NSAIDs have been shown to reduce menstrual blood loss by 30% to 50% in women with menorrhagia. GnRH agonists may be utilized to produce a medical menopause. However, they are expensive, and long-term usage is associated with significant side effects (e.g., osteoporosis). Hysterectomy is the definitive treatment for leiomyomata in symptomatic women who have completed childbearing. However, the mortality associated with hysterectomy is approximately 0.5/1,000, at that age. It is generally reserved for women who have failed medical management or have symptoms or signs related to fibroid size. Of particular note, levonorgestrel-releasing IUD can be used as an alternative. Studies have shown a reduction in uterine volume and bleeding and an increase in hematocrit after placement of IUD. If the fibroids are intracavitary, an IUD should NOT be used.

You start an NSAID. She responds moderately well. You add an OCP, which adequately controls her symptoms. If the patient had not responded to medical therapy, you might have considered referral to a gynecologist for potential surgical therapy.

Question 15.9.5 What other characteristic(s) would have prompted evaluation for surgery?

A) Prolapsing fibroid through the cervix

B) 5-cm submucosal myoma protruding 50% into the uterine cavity

C) Rapidly enlarging uterus

D) 20-week sized uterus and pelvic pressure

E) All of the above

Answer 15.9.5 The correct answer is "E." A fibroid prolapsing through the cervix has a potential for necrosis and infection and may require surgical removal. A submucosal myoma, especially one that distorts the uterine cavity, can contribute to menorrhagia. A rapidly enlarging uterus would be concerning for a uterine malignancy and would require further investigation. Larger uterine fibroids are more likely to contribute to symptoms of pelvic pressure and pain, which may only respond to surgical correction.

The patient returns in 1 year for her annual examination, and her pelvic examination is unchanged. However, she expresses concern that these fibroids could become cancerous.

Question 15.9.6 What is the risk of uterine malignancy (leiomyosarcoma) in a patient with fibroids?

A) <1%

B) 5% to 10%

C) 40% to 50%

D) 90% to 95%

Answer 15.9.6 The correct answer is "A." The estimated incidence of leiomyosarcoma discovered at the time of surgery for fibroids is less than 1%. A leiomyosarcoma is a malignant tumor that does not arise from preexisting benign leiomyoma. Leiomyosarcomas typically arise in the fifth or sixth decade of life and are usually associated with abnormal bleeding or a rapidly enlarging uterus.

Objectives: Did you learn to…

  • Recognized contraindications to IUD use?

  • Treat menorrhagia?

  • Describe management principles for uterine leiomyomas?

A 20-year-old nulligravid female presents for evaluation of irregular menstrual cycles. Her past medical history is uncomplicated. Her gynecologic history is remarkable for menarche at age 12 years, irregular menses occurring every 21 to 40 days, and lasting 5 days, with last menstrual period 6 months ago. Her examination reveals no abnormal hair growth, no acne, normal Tanner stage V breast development, normal external genitalia and cervix, with a slightly enlarged uterus at 10 weeks size, and no adnexal masses.

Question 15.10.1 What is the first test you should order?

A) Pelvic ultrasound

B) Serum prolactin level

C) Urine hCG

D) Serum FSH

E) Serum TSH

Answer 15.10.1 The correct answer is "C." Primary amenorrhea is defined as either (1) the absence of menarche by 15 years with secondary sexual characteristics or (2) the absence of menarche by 13 years of age in the absence of secondary sexual characteristics. Secondary amenorrhea is defined as the absence of periods for 3 cycles in a woman who previously had menses. Pregnancy is the most common cause of secondary amenorrhea and thus must always be ruled out. After pregnancy has been excluded, the diagnostic focus is on differentiating between anatomic cause, ovarian failure, and endocrine abnormalities. The other tests may be done at some point, but a pregnancy test is first.

The urine pregnancy test is positive. Because of her irregular menses, you schedule her in 1 week for a vaginal ultrasound to confirm her gestational age. However, she calls your nurse in 3 days with vaginal spotting and lower pelvic cramping.

Question 15.10.2 The most important diagnosis to confirm or exclude is:

A) Spontaneous abortion

B) Incomplete abortion

C) Inevitable abortion

D) Ectopic pregnancy

E) Twin gestation

Answer 15.10.2 The correct answer is "D." Ectopic pregnancy is the leading cause of pregnancy-related death during the first trimester in the United States. Risk factors for ectopic pregnancy include prior PID, prior ectopic pregnancy, and prior tubal or other pelvic surgery. Spontaneous, incomplete, inevitable, threatened, and missed abortions (or miscarriages) are terms to describe pregnancy loss occurring before 20 weeks of gestation. Each can result in significant maternal morbidity (including hemorrhage and infection) or death. However, these outcomes are less frequent than with an ectopic pregnancy.

Question 15.10.3 Which of the following tests is/are important in the management of this patient?

B) Serum quantitative β-hCG

C) Hematocrit, blood type, and screen

D) Pelvic ultrasound

E) All of the above

Answer 15.10.3 The correct answer is "E." "A," progesterone level, helps to assess the viability of the pregnancy. A progesterone level <5 ng/mL is non-reassuring (85% spontaneous abortion, 14% ectopic pregnancy), and a level >25 ng/mL is reassuring (<2% ectopic pregnancies). Two serum quantitative β-hCGs obtained 48 hours apart help assess viability. A rise in level of >66% will occur in 85% of normal pregnancies, but only 17% of ectopic pregnancies. Transvaginal ultrasounds should be able to detect an intrauterine pregnancy at hCG levels of 1,500 to 2,000 IU. With hCG levels below 1,500 IU, an ultrasound is often still helpful for identifying an adnexal mass and potential ectopic pregnancy. A blood type and screen and hematocrit are important to evaluate for Rh status and anemia and prepare for transfusion should the hemorrhage be significant.

Upon presentation, the patient has normal vitals and appears clinically stable. The blood work returns with a hematocrit of 38% and blood type A negative, antibody screen positive for anti-D. The hCG level is 5,500 IU and progesterone is 18 ng/mL. She has not received any treatment for this bleeding prior to coming to your office.

Question 15.10.4 While waiting for her ultrasound, you counsel her regarding her blood work, which indicates:

A) She should receive RhoGAM immediately, given that she is Rh negative

B) She has previously been exposed to the D antigen and has developed antibodies

C) This fetus is D antigen positive and she has developed antibodies to this fetus

D) She should receive RhoGAM only after confirming that she has an intrauterine pregnancy

Answer 15.10.4 The correct answer is "B." Rh "negative" or "positive" refers to the D antigen, the antigen that is responsible for most cases of Rh sensitization. RhoGAM, or anti-D immune globulin, is used to prevent the development of Rh-D antibodies. If Rh-D antibodies are already present (i.e., the patient has already been sensitized), RhoGAM is not effective. This is why options "A" and "D" are incorrect. This patient has evidence of antibodies and has previously been exposed to the Rh-D antigen. "C" is incorrect because not enough time has elapsed with her current bleeding to have induced antibody production to the D antigen. Without RhoGAM, 17% of mothers become sensitized. RhoGAM is recommended with ectopic pregnancies, spontaneous abortions, induced abortions, threatened abortions, amniocentesis, antepartum hemorrhage, and routinely at 28 weeks of gestation for Rh-D-negative women who are not already sensitized.

When Mrs Kline is 2 days postpartum The nurse would expect fundal height to be?
HELPFUL TIP:

If the Rh-D-negative woman is positive for D antibody, either she has previously been exposed to the D-antigen and mounted a response or she was given anti-D immune globulin during the previous 12 weeks (and it is still present).

The patient returns after her ultrasound, which reveals an intrauterine pregnancy measuring 6 weeks, 5 days gestation with cardiac activity at 110 beats per minute. The laboratory staff called you to let you know that the antibody screen was in error, and in fact the patient is A negative, antibody screen negative. Therefore, you administer RhoGAM.

Question 15.10.5 You counsel the patient that her diagnosis at this time is:

A) Threatened abortion

B) Missed abortion

C) Complete abortion

D) Ectopic pregnancy

E) All remain in the differential

Answer 15.10.5 The correct answer is "A." Threatened abortion is defined as any vaginal bleeding in the first trimester or up to 20 weeks of gestation, which accompanies a currently viable intrauterine pregnancy with a closed cervix. A missed abortion is retention of dead products of conception in utero. A complete abortion indicates the pregnancy and all products of conception have passed from the uterus.

Over the next 2 days, her cramping and bleeding increase and she passes tissue (complete abortion). The patient is somewhat anxious regarding her fertility and is concerned that she may have something "wrong" with her that led to the miscarriage.

Question 15.10.6 What is the most likely etiology of this miscarriage?

A) Uterine anomalies

B) Maternal infection

C) Undiagnosed maternal diabetes

D) Embryonic chromosomal abnormality

Answer 15.10.6 The correct answer is "D." Approximately 50% of first trimester spontaneous miscarriages are due to chromosomal abnormalities. Some of these occur prior to clinical recognition of the pregnancy and will go unrecognized by the patient. All of the others can be responsible for a first-trimester miscarriage as well but are less common.

When Mrs Kline is 2 days postpartum The nurse would expect fundal height to be?
HELPFUL TIP:

The rate of spontaneous abortion increases with advancing maternal age. For women younger than 20 years with recognized pregnancies, the rate of spontaneous abortion is about 12%; the rate increases to 26% in women older than 40 years.

Question 15.10.7 The next step in the care of this patient is:

A) Hysterosalpingogram

B) Gonorrhea and chlamydia cultures

C) Glucose screening

D) Paternal chromosome testing

E) Counseling and reassurance

Answer 15.10.7 The correct answer is "E." Evaluation to determine the cause of this patient's pregnancy loss is not recommended. In general, evaluation is not recommended for a single first-trimester spontaneous loss if the woman is otherwise healthy. Patients with 2 to 3 consecutive spontaneous pregnancy losses are candidates for an evaluation to determine the etiology.

Objectives: Did you learn to…

  • Maintain a high degree of suspicion for pregnancy in a patient presenting with secondary amenorrhea?

  • Identify causes of first-trimester bleeding?

  • Define terminology used in spontaneous abortion (e.g., missed, completed, and threatened)?

  • Manage early pregnancy loss?

  • Describe Rh-D isoimmunization?

You are now assuming care of a 28-year-old G3 P2002 at 38 3/7 weeks. Her pregnancy has been uncomplicated. On examination, you note her blood pressure is 102/66 mm Hg, urine dipstick is negative for protein and glucose, fetal heart tones are 154 beats per minute, and the fundal height is 44 cm. In reviewing her antepartum record, you note that her previous babies weighed 8 lb 8 oz and 9 lb 6 oz at delivery. She has no history of shoulder dystocia. During this pregnancy, her lab results have been normal, and her 1-hour glucose after 75 g of glucose was 127 mg/dL at 28 weeks.

Question 15.11.1 Noting that she has a size–date discrepancy, what is your next step in the evaluation and management of this patient?

A) 3-hour glucose tolerance test

B) Nonstress test

C) Fetal ultrasound for growth and amniotic fluid index

D) Immediate induction of labor

E) Contraction stress test (CST)

Answer 15.11.1 The correct answer is "C." Size–date discrepancy can be caused by numerous maternal–fetal factors. Given the patient's history, the likely etiologies are either fetal macrosomia or polyhydramnios. Additionally, her body habitus could render the fundal height measurement inaccurate (her weight is not noted). To best determine the etiology, an ultrasound with amniotic fluid index is warranted. A 3-hour glucose tolerance test ("A") is not indicated since her screening glucose tolerance test was normal. A nonstress test ("B") may be warranted depending on the outcome of the fetal ultrasound and amniotic fluid index. However, if growth and fluid were normal, a nonstress test would not be indicated. The CST ("E") is utilized to assess fetal well-being in utero. A CST is done by administering oxytocin to induce contractions and observing the resulting FHR tracing. All else being equal, a CST is not warranted at this time.

When Mrs Kline is 2 days postpartum The nurse would expect fundal height to be?
HELPFUL TIP:

Controversy surrounds the issue of induction for fetal macrosomia. Induction for macrosomia has not been shown to improve maternal or fetal outcomes.

Question 15.11.2 Risk factors for fetal macrosomia include all of the following EXCEPT:

A) Gestational age

B) Maternal smoking

C) Excessive maternal weight gain

D) Multiparity (not a multiple gestation)

E) Macrosomia in a prior infant

Answer 15.11.2 The correct answer is "B." Maternal smoking is associated with restricted fetal growth. All of the others are associated with macrosomia. Additionally, male fetus and high maternal birth weight, and bad karma in a previous life are associated with macrosomia.

Ultrasound findings demonstrate a fetus in the vertex presentation with an estimated fetal weight of 4,200 g (9 lb 2 oz) and an amniotic fluid index of 12.6 cm (normal). You check her cervix and note that she is 1 cm dilated, 50% effaced, with vertex at 0 station.

Question 15.11.3 The optimal management at this time is:

A) Induction of labor

B) Cesarean section

C) Expectant management

D) Repeat the ultrasound weekly

E) Initiate a weight loss program

Answer 15.11.3 The correct answer is "C." Fetal macrosomia is generally defined as a birth weight greater than 4,500 g. Large for gestational age implies a birth weight greater than the 90th percentile for a given gestational age (or 4,000 g at delivery). Expectant management with spontaneous labor onset generally has been shown to have the best outcomes; all interventions entail an increase in fetal and maternal morbidity.

When Mrs Kline is 2 days postpartum The nurse would expect fundal height to be?
HELPFUL TIP:

According to ACOG (2009), prophylactic cesarean section should be offered for estimated fetal weight >5,000 g in nondiabetic pregnancies and >4,500 g in diabetic pregnancies.

When Mrs Kline is 2 days postpartum The nurse would expect fundal height to be?
HELPFUL TIP:

Maternal risk from macrosomia is primarily related to labor abnormalities and includes postpartum hemorrhage, significant vaginal lacerations, cesarean delivery, and infection.

Contractions start spontaneously. Shortly after arrival to labor and delivery at 39 weeks, your patient has spontaneous rupture of membranes with clear fluid and requests an epidural for management of labor pain. Her cervix is 5 cm dilated, 90% effaced, and 0 station. FHR baseline is 145 bpm with moderate variability and no decelerations (a Category I tracing). Four hours later, she is comfortable after epidural is placed. Her cervix is unchanged.

Question 15.11.4 Appropriate intervention at this time is:

A) Induction of labor

B) Augmentation of labor

C) Cesarean section

D) Expectant management

E) Ultrasound to confirm vertex presentation

Answer 15.11.4 The correct answer is "B." Her labor has suffered from arrest of dilation and descent. At this time, the appropriate management is augmentation of labor. Induction of labor is the technical term for initiating uterine contractions before the onset of spontaneous labor, so "A" is incorrect. While expectant management may be acceptable at this point after counseling the patient, there is a risk of prolonged rupture of membranes and chorioamnionitis. Cesarean section at this point would be premature and unnecessary.

With IV oxytocin, she proceeds to complete and after 2 hours of pushing is ready to deliver. You anticipate a shoulder dystocia.

Question 15.11.5 Appropriate maneuvers to reduce a shoulder dystocia include all of the following EXCEPT:

A) McRoberts maneuver (flexing the patient's knees up against the abdomen)

B) Suprapubic pressure

C) Fundal pressure

D) Delivery of the posterior arm

E) Woods corkscrew

Answer 15.11.5 The correct answer is "C." Fundal pressure may further worsen impaction of the shoulder and may result in uterine rupture—both are generally considered bad outcomes. All the others may be used to help relieve shoulder dystocia. To manage dystocia, the Advanced Life Support for Obstetrics (ALSO) course uses the "HELPERR" mnemonic: H—call for help (wait … aren't you the help?), E—evaluate for episiotomy, L—legs flexed at the hip and knee (McRoberts maneuver), P—suprapubic pressure, E—enter to rotate fetus (Rubin II, Woods corkscrew maneuver, reverse corkscrew maneuver), R—remove posterior arm, and R—roll the patient. There is no evidence that any one maneuver is superior to another in releasing an impacted shoulder or decreasing the chance of injury. Typically, McRoberts maneuver is used first. Last resort techniques include intentional fracture of the fetal clavicle, cephalic replacement (aka, the Zavanelli maneuver—pushing the head back up into the pelvis and performing a cesarean section), and transcutaneous symphysiotomy. Why obstetricians continue to use confusing eponyms is beyond us. But if you want to belong to the club you need to know the secret passwords ….

When Mrs Kline is 2 days postpartum The nurse would expect fundal height to be?
HELPFUL TIP:

Risk factors for shoulder dystocia include: previous shoulder dystocia, gestational diabetes, post-dates pregnancy, maternal short stature, abnormal pelvic anatomy, suspected fetal macrosomia, protracted active phase of first stage of labor, protracted second stage of labor, and assisted vaginal delivery.

In the same way that carrying an umbrella prevents a thunderstorm, merely being prepared for dystocia obviates the need for special maneuvers. Happy day! Time to talk to her about nursing.

Question 15.11.6 You tell her the benefits of breastfeeding, which include all the following EXCEPT:

A) Species-specific and age-specific nutrients for infants

B) Adequate iron for premature newborns

C) High level of immune protection from colostrum

D) Decreased risk of breast and ovarian cancer in mothers who breastfeed

E) Fewer illnesses while the infant is breastfed

Answer 15.11.6 The correct answer is "B." The benefits of breastfeeding for infants, women, families, and society are well documented. However, human milk may not provide adequate iron for premature newborns, term infants whose mothers have low iron stores, and infants older than 6 months. All the other choices are correct.

When Mrs Kline is 2 days postpartum The nurse would expect fundal height to be?
HELPFUL TIP:

Breast milk should only be kept at room temperature for 6 hours. However, it is safest to refrigerate it for 4 days maximum or freeze immediately. Do not heat refrigerated breast milk in the microwave, as it will destroy valuable micronutrients. Warming in hot water is a better way of reheating.

Your patient and her infant do well and are discharged home on postpartum day 2. She returns for her postpartum examination in 6 weeks. She notes incontinence of urine several times a day, especially with coughing, laughing, and sneezing. She has not experienced nocturia, dysuria, or hematuria.

Question 15.11.7 These symptoms are indicative of which type of incontinence:

A) Stress incontinence

B) Urge incontinence

C) Overflow incontinence

D) Functional incontinence

E) Psychogenic incontinence

Answer 15.11.7 The correct answer is "A." Stress incontinence is the involuntary loss of urine during physical activity such as coughing, laughing, jumping, running, and sneezing. Urge incontinence is the involuntary loss of urine associated with an abrupt and strong desire to void (detrusor overactivity). Overflow incontinence is the involuntary loss of urine due to under-activity of the detrusor muscle (e.g., neurogenic bladder) or obstruction (e.g., BPH in men) but this (urinary outlet obstruction, not BPH) is becoming more common in women. Functional incontinence is loss of urine associated with physical limitations (e.g., mobility restriction, arthritis, and dementia) in persons who have otherwise adequate bladder control. Psychogenic incontinence, incontinence secondary to severe depression or other psychological problem, is a rare disorder in this population and should be considered a diagnosis of exclusion. It is more frequent in patients with dementia.

Question 15.11.8 Which of the following is most important prior to initiating incontinence therapy?

A) Evaluate for a vaginal fistula using a dye instillation test

B) Obtain urinalysis followed by urine culture if abnormal

C) Refer for cystoscopy

D) Order urodynamic studies

Answer 15.11.8 The correct answer is "B." The first step in the diagnosis of incontinence is a detailed history and physical examination. A urinalysis and urine culture should be obtained to exclude urinary tract infection. Bacteriuria should be treated because the endotoxin produced by Escherichia coli may trigger abnormal detrusor activity or act as an α-adrenergic blocker (cool, huh?). Additional diagnostic tools may include a voiding diary, stress test (have the patient cough or bear down and see if there is leakage from the urethra), checking a postvoid residual, cystometry, and cystourethroscopy. If there is concern for a vaginal fistula elicited by the history and physical examination, dye instillation testing may be warranted (IV dye to evaluate for a ureteral fistula, bladder instillation to evaluate for a bladder fistula).

When Mrs Kline is 2 days postpartum The nurse would expect fundal height to be?
HELPFUL TIP:

Bariatric surgery (gastric banding, etc.) reduces the rate of urinary incontinence in women who qualify: body mass index (BMI) >40 or BMI >35 + comorbidities (DM, sleep apnea, severe joint disease, weight-related cardiomyopathy).

When you have completed your evaluation, you determine that she has stress urinary incontinence.

Question 15.11.9 The best initial treatment option is:

A) Pelvic muscle exercises

B) Trial of oxybutynin hydrochloride

C) Fitting of a pessary

D) Surgical consult

E) Incontinence pads

Answer 15.11.9 The correct answer is "A." Pelvic muscle exercises (e.g., Kegel exercises) facilitate improved urinary control in 40% to 75% of patients. The correct method can be taught during a routine pelvic examination. Pelvic physical therapy can be used to provide feedback on the patient's success with these exercises. "B" is incorrect. Oxybutynin hydrochloride is approved for detrusor instability (i.e., urge incontinence) and does not appear to be effective for stress incontinence. It also has significant anticholinergic side effects. "C" is incorrect. Several vaginal pessaries have been designed with the intent of providing differential support to the urethrovesical junction for treatment of stress urinary incontinence. This is an option for many women, especially those who want to avoid surgery, but a pessary would not be the initial treatment. "D" is incorrect. Surgical correction is typically reserved until 6 to 12 months postpartum, as the symptoms may continue to improve during that time. Note that surgical mesh has a very high rate of complications when used for the treatment of incontinence (mid-urethral sling) and, according to the FDA, has NO benefit over the traditional surgical treatment of stress incontinence. We would not subject our relatives to this …

Objectives: Did you learn to…

  • Evaluate and manage fetal macrosomia?

  • Manage a delivery complicated by shoulder dystocia?

  • Discuss the benefits of breastfeeding?

  • Classify urinary incontinence and treat stress-type incontinence?

A 31-year-old nulligravid single female presents for an annual examination. She has no gynecologic concerns, and her last menstrual period was 3 weeks ago. She uses oral contraception and has regular cycles. Her examination is unremarkable. The Pap smear returns with high-grade squamous intraepithelial neoplasia (HSIL).

Question 15.12.1 You notify her of the Pap smear findings and explain this indicates:

A) She has cervical cancer

B) She needs further diagnostic testing including colposcopy and possible biopsy

C) She needs treatment with cryotherapy or laser ablation

D) She should have the Pap smear repeated in 3 to 4 months

E) She should be enrolled in a hospice program

Answer 15.12.1 The correct answer is "B." Cervical cytology is the most effective cancer-screening program ever implemented. Both types of screening, liquid based and traditional Pap smears, have the same accuracy, and both types are endorsed for screening. Abnormal Pap smear findings require further investigation with colposcopy and directed biopsy, if indicated. Given the finding of high-grade intraepithelial neoplasia on the Pap smear, repeating the Pap smear in 3 months is inappropriate. An endocervical specimen (Pap smear or endocervical curettage) will need to be performed regardless of the appearance of the ectocervix to exclude endocervical pathology. Treatment with a nonexcisional procedure ("C") is not acceptable and premature without a biopsy to confirm the diagnosis first.

You perform a colposcopy, and the cervix appears grossly normal. An endocervical Pap smear is obtained. There are acetowhite changes with areas of mosaicism at the squamocolumnar junction from the 4 o'clock to 11 o'clock position. The colposcopy is adequate.

Question 15.12.2 When you discuss the cervical findings you explain that:

A) She needs a biopsy of the abnormal area

B) Given the previous Pap smear, you recommend a "see and treat" loop electrosurgical excision procedure (LEEP)

C) The findings are consistent with dysplasia; no further therapy is warranted

D) She needs a repeat Pap smear in 3 to 4 months

E) Either A or B would be acceptable choices

Answer 15.12.2 The correct answer is "E." When acetic acid is applied to the cervix, abnormal cells tend to turn white ("acetowhite"), and as the acetowhite fades, the degree of vascularity can be appreciated. Mosaicism, or mosaic changes, refer to areas with increased vascularity and are indicative of abnormal changes, often dysplasia. However, "visual" findings at colposcopy are no more than suggestive, so "C" is incorrect. Biopsy of the area is warranted to obtain a tissue diagnosis. The see-and-treat approach ("B") is an option for our patient since she is 31 years old; however the see-and-treat approach should not be done in young women between the ages of 21 and 24 per the current ASCCP guidelines. In this age range lesions often regress spontaneously. However, consider the "see and treat" approach in any patient who is unlikely to follow up.

The endocervical Pap smear is negative for dysplasia. You also performed a biopsy of the acetowhite change area, which revealed high-grade cervical intraepithelial neoplasia (CIN III).

Question 15.12.3 Which of the following do you recommend?

A) Hysterectomy

B) Trachelectomy

C) Cytological follow-up

D) LEEP or LASER ablation

E) 5-fluorouracil intravaginally

Answer 15.12.3 The correct answer is "D." While some patients with severe dysplasia (CIN III) may be candidates for a hysterectomy, it would be an overly aggressive approach in this 31-year-old female who has never been pregnant and may wish to conceive in the future. Additionally, women who had high-grade CIN before hysterectomy can develop recurrent vaginal dysplasia (VaIN) or cancer at the vaginal cuff. Likewise, trachelectomy (removal of the entire cervix) would be overly aggressive with no added benefit over a LEEP procedure. Cytological follow-up or "expectant management" is NOT recommended for high-grade dysplasia in this patient. The likelihood of regression is low in this patient's age demographic, while the likelihood of persistence or progression to cancer is unacceptably high. Young women between 21 and 24 do have the option proceeding with treatment (which is preferred with CIN III or inadequate colposcopy) OR observation with cytology and colposcopy at 6 month intervals for 12 months. A LEEP or ablation therapy would be the best recommendation in this patient. The use of 5-fluorouracil intravaginally is typically not recommended for initial treatment of cervical dysplasia. It is used instead for vaginal dysplasia, when the extent of disease precludes complete excision or destruction.

Your patient wants to know the likelihood of regression to "normal" without treatment.

Question 15.12.4 What will you counsel?

A) About 90% of CIN III lesions will become invasive cancer without aggressive treatment

B) About one-third of CIN III lesions spontaneously regress

C) About two-thirds of CIN III lesions spontaneously regress

D) About 90% of CIN III lesions will remain CIN III on follow-up after 10 years

Answer 15.12.4 The correct answer is "B," although the data on prognosis varies due to variation in histologic diagnoses of CIN. In some studies, about one-third of untreated CIN III lesions will spontaneously regress, while about 50% will persist and the remaining will progress to invasive carcinoma. Other studies have shown progression to invasive carcinoma much higher.

She is concerned about future childbearing if she undergoes a LEEP.

Question 15.12.5 All of the following are possible complications of LEEP EXCEPT:

A) Cervical incompetence

B) Cervical stenosis

C) Cervical ectopic pregnancy

D) Decreased fertility

E) Premature rupture of membranes

Answer 15.12.5 The correct answer is "C." Cervical incompetence, stenosis, decreased fertility, and premature rupture of the membranes have been identified following all types of cone procedures, including LEEP, and are estimated to occur following less than 1% of procedures. Each of these complications seems to be related to the volume of tissue removed with the procedure rather than the procedure itself. Cervical ectopic pregnancy is quite rare and has NOT been associated with cone or LEEP procedures.

She undergoes LEEP treatment as recommended. The pathology reveals CIN III with margins uninvolved.

Question 15.12.6 What do you counsel her about follow-up?

A) She should return for a Pap smear and pelvic examination in 1 year

B) She should return for a Pap smear plus HPV co-testing in 12 and 24 months

C) She should return for a Pap smear in 2 to 3 months

D) She should return for a colposcopy and Pap smear in 2 to 3 months

E) She should just "chill out"; no follow-up is indicated

Answer 15.12.6 The correct answer is "B." The current recommendations for follow-up include a Pap smear plus HPV co-testing at 12 and 24 months. If both are negative, HPV co-testing should then be repeated 3 years later and if all remains negative she then may return to routine age-based screening for 20 years. If any of the Pap smears reveal atypical squamous cells (ASC) or higher grade dysplasia, or positive high-risk HPV, the patient should undergo repeat colposcopy.

At the follow-up evaluation at 12 months, her cervix appears normal, without lesions or discharge. The Pap smear returns as normal with negative high-risk HPV co-testing; however, testing is limited by absence of endocervical cells on the Pap smear.

Question 15.12.7 What is the best recommendation for management of this patient now?

A) Repeat Pap smear and co-testing in another 12 months

B) Cervical dilation and endocervical curettage

C) Cervical dilation and endocervical Pap smear

D) Repeat LEEP

E) Repeat the Pap smear as soon as possible

Answer 15.12.7 The answer is "A." Since the HPV test was negative there is no need for further testing at this time and screening should proceed as previously outlined above for her CIN III. However, further evaluation now WOULD be necessary if the HPV was positive or unknown.

All your partners are on vacation and you are covering all their patients' tests. Suddenly you receive a STAT page from the pathology lab (a pathological emergency?). They wanted to notify you that they made a mistake when interpreting one of the Pap smear results on a 33-year-old patient and, in fact, the correct report should have been ASC-US (atypical squamous cells of undetermined significance) instead of ASC-H (atypical squamous cells, high grade). The lab technician is new here and is not quite sure if they can add on an "HPV test" to the Pap sample.

Question 15.12.8 As you prepare yourself to call the patient, what do you tell her is the next step?

A) Repeat Pap smear in 1 year

B) Ask the lab personnel to investigate if the HPV test can be done on that sample

C) Contact your malpractice attorney

D) Both A and B

Answer 15.12.8 The correct answer is "D," according to current ASCCP guidelines. If you are able to do HPV co-testing and it returns negative she could have repeat co-testing in 3 years. If you choose to repeat cytology only in 1 year and it returns normal, she may return to age-appropriate screening. If the cytology returns with any other abnormalities greater than or equal to ASC then a colposcopy is indicated. Colposcopy would also be indicated if the HPV test returns positive. Make sure that the Pap report does not actually read ASC-H (atypical squamous cells: cannot exclude high-grade SIL). In this case, colposcopy would be indicated no matter the HPV status in this patient's age demographic. If the patient was pregnant, or between the ages of 21 and 24, management would be different. Making sure your malpractice insurance is current and up to date is always a good idea but probably is unnecessary in this case.

When Mrs Kline is 2 days postpartum The nurse would expect fundal height to be?
HELPFUL (MAYBE) TIP:

The guidelines for Pap smears are ridiculously complex and include differences based on patient age, pregnancy status, etc. We strongly recommend that you review the ASCCP guidelines and algorithms available at http://www.asccp.org/consensus.shtml.

Objectives: Did you learn to…

  • Manage an abnormal Pap test?

  • Recognize the indications for colposcopy?

  • Evaluate and manage cervical dysplasia?

  • Recognize the risks and potential complications of LEEP?

  • Manage the absence of endocervical cells on a Pap test?

You are called to the emergency department to evaluate a patient with a 2-day history of abdominal pain. She is a 24-year-old G1 P1 female whose LMP was 1 week ago. On the "1–10" scale, her pain is a "12." She is on oral contraceptives for birth control. She has "never missed a pill" and "could not possibly be pregnant." Her pain is across her lower abdomen and a little more on the right side than the left. She has felt feverish. She has had some nausea but no vomiting. She denies bowel or bladder problems. Her pain improves with acetaminophen and worsens with activity.

On examination, she appears uncomfortable but not toxic. Her temperature is 38°C, but the rest of her vitals are normal. Her abdominal examination reveals decreased bowel sounds, with tenderness to palpation primarily across the lower quadrants. She has minimal guarding and no rebound tenderness. Her pelvic examination is remarkable for cervical motion tenderness. The uterus is of normal size and consistency with no masses.

Question 15.13.1 Which of the following diagnoses can be absolutely excluded from your differential at this point?

A) Ectopic pregnancy

B) Appendicitis

C) Pelvic inflammatory disease (PID)

D) Pyelonephritis

E) None of the above diagnoses should be excluded based on the information available

Answer 15.13.1 The correct answer is "E." The differential for lower abdominal pain in a young female includes all of the above and more. Even though your patient seems unlikely to be pregnant due to her consistent use of contraceptives and recent menses, you should not exclude pregnancy without a negative urine hCG. Emergency department studies consistently show that women who claim to never have been sexually active can be pregnant. Either something metaphysical is going on here or history cannot always be trusted.

You obtain cultures/PCR for chlamydia and gonorrhea. The urine pregnancy test is negative ("I told you not to waste health care dollars—especially in this economy," your patient complains). The urinalysis is negative for nitrites and leukocytes, and the WBC is 15,600/mm3 with an increase in bands. She reports that she's had an appendectomy.

Question 15.13.2 What is the most appropriate next step?

A) Consult surgery and gynecology to confirm your findings

B) Admit for IV antibiotics and IV hydration

C) Treat as an outpatient with antibiotics and schedule follow-up for 36 to 48 hours

D) Treat with IV antibiotics on an outpatient basis utilizing visiting nurse care

E) Obtain cultures, discharge the patient, and treat based on culture results

Answer 15.13.2 The correct answer is "C." The patient's history, examination, and diagnostic tests are most consistent with pelvic inflammatory disease (PID). PID is a clinical syndrome caused by the ascent of microorganisms from the lower genital tract (e.g., vagina) to the upper genital tract (e.g., endometrium). Most cases of PID can be managed in the outpatient setting. Indications for hospitalization are listed in Table 15-5.

TABLE 15-5CRITERIA FOR ADMISSION FOR THE TREATMENT OF PID

View Table||Download (.pdf)

TABLE 15-5 CRITERIA FOR ADMISSION FOR THE TREATMENT OF PID

  • Uncertain diagnosis

  • Surgical emergencies (e.g., appendicitis) cannot be excluded

  • Suspected tubo-ovarian abscess

  • Concurrent pregnancy (due to high risk of maternal mortality, fetal wastage, and preterm delivery)

  • Severe illness, intractable nausea and vomiting, or high fever

  • Patient cannot tolerate or follow an outpatient regimen (e.g., severe vomiting)

  • Lack of clinical response to oral outpatient antimicrobial therapy

When Mrs Kline is 2 days postpartum The nurse would expect fundal height to be?
HELPFUL TIP:

The diagnosis of PID is a clinical one and not laboratory based! Untreated PID has significant morbidity and mortality; empiric treatment is recommended if the patient meets the following minimal diagnostic criteria:

  • Uterine/adnexal tenderness or

  • Cervical motion tenderness and

  • No other cause for illness identified

Other helpful (but not necessary) criteria include:

  • Temperature ≥38.3°C

  • Abnormal cervical or vaginal mucopurulent discharge

  • Presence of an abundant number of white cells on saline microscopy of vaginal fluid

  • Adnexal mass

  • Laboratory evidence of gonorrhea or chlamydia infection

  • Elevated C-reactive protein and/or ESR

Question 15.13.3 For empiric antibiotic therapy for PID in this patient, you prescribe:

A) Amoxicillin 500 mg PO TID for 14 days

B) Ceftriaxone 250 mg IM once PLUS azithromycin 1 g

C) Ceftriaxone 250 mg IM once PLUS doxycycline 100 mg PO BID for 14 days

D) A and B

E) B and C

Answer 15.13.3 The correct answer is "C." Recommendations for treatment of PID include ceftriaxone (Rocephin) 250 mg IM once PLUS doxycycline 100 mg PO BID for 14 (yes, 14) days. Metronidazole may be added to this regimen to cover anaerobic bacteria. Note that single-dose azithromycin is not indicated for the treatment of PID, only for cervicitis. Thus, 14 days of doxycycline are indicated.

The current (2015) CDC treatment guidelines for rectal, oral, cervical, or penile gonorrhea include ceftriaxone 250 mg IM once PLUS azithromycin 1 g or 7 days of doxycycline. This is true even if the patient has ONLY gonorrhea. This is to prevent the development of resistant gonorrhea and also due to the high co-infection rate of gonorrhea with chlamydia. If ceftriaxone is not available, cefixime 400 mg orally in a single dose PLUS azithromycin 1 g orally in a single dose is an appropriate alternative. Of note, patients who have been treated for gonorrhea should be re-tested 3 months after treatment regardless of whether they believe their sex partners were treated as there is a high incidence of reinfection (CDC, 2015).

Just when you were about to discharge her the patient starts vomiting and is unable to keep down her oral medications (doxycycline, acetaminophen). You admit her and she is discharged when she is tolerating oral intake and has been afebrile for 48 hours. You instruct her to finish a 14-day course of doxycycline. She presents for follow-up a week later, at which time her symptoms have completely resolved.

Question 15.13.4 Which of the following is a potential consequence of PID?

A) Infertility

B) Chronic pelvic pain

C) Increased risk for ectopic pregnancy

D) Recurrent PID

E) All of the above

Answer 15.13.4 The correct answer is "E." All the choices above are potential sequelae of PID. Additionally, a tubo-ovarian abscess may develop.

When Mrs Kline is 2 days postpartum The nurse would expect fundal height to be?
HELPFUL TIP:

The etiology of PID is polymicrobial, although sexually transmitted infections, predominantly gonorrhea and/or chlamydia, are implicated in up to two-thirds of cases. Antibiotic regimens are chosen for broad coverage. The CDC no longer recommends the use of fluoroquinolones for treatment of PID or gonorrhea due to increasing resistance. Of note, there is ceftriaxone-resistant gonorrhea in Japan with intermediate-resistant gonorrhea in the United States.

You see her next for her annual examination. She quit taking her oral contraceptive 3 months ago after separating from her husband (the jerk gave her chlamydia, after all!). Subsequently, her periods have been irregular. She is not obese, has minimal acne, and no hirsutism or galactorrhea. Her physical examination is essentially normal. Urine hCG is negative (you did immediately think of an hCG, didn't you?) and a serum TSH is normal.

Question 15.13.5 What is the most likely etiology of the irregular cycles?

A) Anovulation

B) Pituitary tumor

C) Polycystic ovarian syndrome

D) Premature ovarian failure

E) Androgen secreting tumor

Answer 15.13.5 The correct answer is "A." OCPs work by suppressing ovulation. Resumption of ovulation after pill cessation can take several months (up to 6 months). In the absence of abnormal history, physical examination, or laboratory findings, the other choices are unlikely to be the etiology of the irregular cycles in this patient.

Question 15.13.6 Each of the following is appropriate initial management of this patient's irregular menses, EXCEPT:

A) Expectant management

B) Reestablishment of cycle regulation with OCPs

C) Progestin induced withdrawal cycles

D) Colposcopy with cervical and endometrial biopsies

Answer 15.13.6 The correct answer is "D." Anovulation is expected following OCP cessation; thus, expectant management is a reasonable option, as most people will resume regular cycling within 6 months. She could also opt to resume OCPs for cycle regulation if that is her goal. Withdrawal bleeds could be induced by cyclical progestin challenges. Colposcopy is not indicated in this patient as she has no cytologic abnormalities noted that would require follow-up with colposcopy and biopsy.

The patient is reluctant to resume "the pill." You next evaluate her 8 months later. Her last period was 7 weeks before. A urine pregnancy test is positive. Given her history of PID, you request a quantitative serum β-hCG and are considering a pelvic ultrasound to confirm intrauterine pregnancy.

Question 15.13.7 What is the minimum expected increase in quantitative hCG in early gestation in a normal pregnancy?

A) 20% increase in 24 hours

B) 66% increase in 48 hours

C) 100% increase in 24 hours

D) 10% increase in 48 hours

E) 75% increase in 72 hours

Answer 15.13.7 The correct answer is "B." hCG typically doubles (increases by 100%) every 48 hours in normal gestation. The minimum increase considered to be compatible with a viable pregnancy is 66% in 48 hours.

Question 15.13.8 If the quantitative serum β-hCG is 5,500 ng/mL and the pelvic ultrasound reveals no intrauterine pregnancy but a probable right tubal pregnancy, what would be the most appropriate management option?

A) Medical management with methotrexate

B) Laparoscopic surgery with evacuation of the products of conception

C) Consultation with someone able to do a salpingectomy if necessary

D) Dilation and curettage

E) Hysterectomy

Answer 15.13.8 The correct answer is "C." An ectopic pregnancy may rupture and become a life-threatening event at any point before complete resolution. If the patient becomes hemodynamically unstable, she will need emergent surgery. Thus, an early referral for management is warranted. Medical treatment with methotrexate may be indicated but should be done under close supervision with surgical consultation available.

Question 15.13.9 If the quantitative serum β-hCG had come back at 1,000 ng/mL and no ultrasound were available, how would you have counseled the patient?

A) Given the history of PID, this is most likely an ectopic pregnancy. She should abort the pregnancy at once

B) Given the history of PID, she should remain on bed rest until a definitive diagnosis is made

C) She should be educated about ectopic pregnancy and miscarriage

D) She should have an urgent surgical consultation today for exploratory laparotomy

E) She should read the patient education material entitled "So, You Might Be Having an Ectopic Pregnancy"

Answer 15.13.9 The correct answer is "C." This β-hCG level is consistent with an early gestation given her history of irregular cycles. However, with her history of PID, she is at risk for ectopic pregnancy. Thus, she should receive ectopic pregnancy and miscarriage precautions. While a history of PID increases the risk of ectopic pregnancy by 7- to 10-fold, only 8% of such patients will ever have an ectopic. So, it is still highly probable that the patient has a normal intrauterine pregnancy. If you chose "E," you may also want to hand her the pamphlet entitled "So, Your Doctor Has a Horrible Bedside Manner."

In reality, her quantitative serum β-hCG is 6,000 ng/mL. A pelvic ultrasound confirms a viable intrauterine pregnancy at 8 4/7 weeks of gestation. She wins the ectopic lottery.

When Mrs Kline is 2 days postpartum The nurse would expect fundal height to be?
HELPFUL TIP:

Assisted reproductive technology substantially increases the risk of a heterotopic pregnancy (two pregnancies with only one inside the uterus) to about 1%, and in these patients, the rate of ectopic pregnancy is 4% to 8%, which is about fourfold higher than the general population. Having one ectopic pregnancy predisposes to having a subsequent one.

Objectives: Did you learn to…

  • Evaluate a patient with pelvic pain?

  • Diagnose and treat PID?

  • Evaluate and treat irregular menses?

  • Diagnose and manage ectopic pregnancies?

A 52-year-old female patient of yours presents for an annual examination and Pap smear. Her last menstrual period was 3 months ago. She has intermittent hot flashes and night sweats. Her examination is remarkable for mild vaginal atrophy. She wonders about "estrogen testing" to see if she needs hormone replacement therapy.

Question 15.14.1 How will you counsel her about the role of estrogen testing?

A) Recommend against estrogen testing

B) Recommend buccal swab testing as it is the most accurate

C) Recommend fasting morning serum estradiol testing

D) Recommend estrogen testing with FSH to ensure menopause status

E) Recommend annual estrogen testing after the age of 55 years

Answer 15.14.1 The correct answer is "A." In a woman of the right age with symptoms consistent with menopause, no testing is recommended. Testing is only recommended if the diagnosis is unclear (e.g., a patient younger than 35). It can also be useful when a patient on OCPs presents with symptoms suggestive of menopause. In this case, an FSH could be done after the patient has discontinued hormonal contraception for 1 week (or on the last day of her placebo pills). If the FSH is high (>25 mIU/L), this indicates that the patient has likely entered the menopausal transition.

She jokes that you're probably getting kickbacks from the insurance company for ordering fewer tests, and then she asks, "How will you know if I'm going through menopause without an estrogen level?"

Question 15.14.2 How will you counsel her about menopause and its diagnosis?

A) FSH is the definitive test

B) There is no definitive test of menopause

C) 6 months amenorrhea, elevated FSH, and decreased estradiol confirm the diagnosis

D) 12 months amenorrhea, elevated FSH, and decreased estradiol are the definitive test

E) 2 years of mood swings and hot flashes will clinch the diagnosis

Answer 15.14.2 The correct answer is "B." Menopause is a clinical syndrome characterized by the cessation of spontaneous menstrual periods along with associated symptoms of estrogen deficiency such as hot flashes, vaginal atrophy, and psychological symptoms. As such, there is no definitive test.

She is concerned about menopause and wonders if she needs hormone replacement therapy.

Question 15.14.3 All the following are benefits of estrogen-containing hormone replacement therapy (HRT) EXCEPT:

A) Osteoporosis prevention

B) Decrease in colon cancer risk

C) Decrease in hot flashes and vasomotor symptoms

D) Decrease in stroke risk

Answer 15.14.3 The correct answer is "D." We have learned a lot about HRT in postmenopausal women with data from the Women's Health Initiative. Here's a quick summary. Proven benefits of HRT are limited to the following:

  • Reduced risk of osteoporosis and related fractures.

  • Decreased colon cancer risk (not seen in estrogen only arm). Data now suggest that even though the number of colon cancers is reduced, they are diagnosed at a more advanced stage. HRT is not recommended for chemoprevention of colon cancer in women.

  • Improvement of vasomotor symptoms such as hot flashes.

HRT may increase the risk of the following (variable findings):

  • Breast cancer (estrogen only arm showed a paradoxical reduction in risk of breast cancer)

  • Myocardial infarction (estrogen only arm showed no increase risk)

  • Venous thromboembolic events

  • Stroke

Only women with vasomotor symptoms appear to have improved overall quality-of-life scores with HRT. Given the significant excess risks with HRT usage, many practitioners now recommend short-term HRT use only for vasomotor symptoms and not for osteoporosis or other purported benefits.

Question 15.14.4 An absolute contraindication to use of HRT is:

A) Heart disease

B) Breast cancer

C) Endometrial cancer

D) Previous thromboembolic event

E) All of the above

Answer 15.14.4 The correct answer is "D." Previous thromboembolic disease is the only absolute contraindication to HRT. The others are relative contraindications. While HRT is not routinely recommended for women who have heart disease or a history of breast or endometrial cancer, it may be useful in women who have significant impairment in their quality of life from vasomotor symptoms refractory to other management methods.

You discuss menopause and the potential risks and benefits of HRT with your patient. She decides against HRT for now, but returns in 6 months with continued amenorrhea, hot flashes, and vaginal dryness—which seems to bother her the most.

Question 15.14.5 Options to treat the vaginal dryness include which of the following?

A) Systemic HRT

B) Vaginal estrogen

C) Lubrication

D) All of the above

Answer 15.14.5 The correct answer is "D." Systemic and local estrogen administration are both effective for treating vaginal dryness. Lubrication with vegetable oil and specifically manufactured lubricants can be effective.

Question 15.14.6 How is vaginal atrophy most rapidly and accurately diagnosed?

A) Biopsy of vaginal mucosa

B) Culture of vaginal swab

C) Patient's history and physical examination

D) Serum hormone levels

E) KOH prep of vaginal swab

Answer 15.14.6 The correct answer is "C." The diagnosis of urogenital atrophy is clinical, based on history and physical examination findings. Tests such as microscopic examination of the vaginal smear, will reveal an abundance of basal and parabasal cells and a paucity of mature squamous epithelium. A biopsy is a bit extreme. "B" and "E" are incorrect since atrophy is not an infectious issue.

Despite your tepid endorsement of HRT, she wants to try it to reduce hot flashes. You also counsel her on other options to treat hot flashes.

Question 15.14.7 Further options to treat symptomatic hot flashes include which of the following?

C) Exercise

D) Trial of an SSRI

E) All of the above

Answer 15.14.7 The correct answer is "E." HRT is the most efficacious treatment for vasomotor symptoms with almost 90% of women responding. Other options include progestins such as medroxyprogesterone (Provera) or megestrol (Megace), SSRIs, clonazepam, gabapentin (Neurontin), venlafaxine (Effexor), clonidine, exercise, and environmental modifications (e.g., thermostat settings, fans). Note that we do not list estrogen alone. Unopposed estrogen can increase the risk of endometrial cancer and should be avoided unless the woman has had a hysterectomy. There is conflicting evidence for the efficacy of vitamin E and black cohosh; there is no good data to suggest that propranolol or phytoestrogens, such as soy, are effective in treating vasomotor symptoms.

Objectives: Did you learn to…

  • Evaluate menopausal symptoms?

  • Describe hormone replacement therapy, including risks, benefits, and contraindications for its use?

  • Diagnose and manage atrophic vaginitis?

  • Treat menopausal symptoms?

A 57-year-old postmenopausal patient presents for her annual examination. She is experiencing hot flashes and night sweats, as well as a recurrence of vaginal bleeding. Her medical history is otherwise unremarkable. She wants your opinion about resuming hormone replacement. Her pelvic examination is remarkable for atrophic vaginal mucosal changes, a stenotic cervix without lesions, normal size uterus, no adnexal masses, and no masses palpable on rectovaginal examination.

Question 15.15.1 All of the following are possible causes of her vaginal bleeding EXCEPT:

A) Cervical cancer

B) Uterine polyp

C) Polycystic ovary disease

D) Atrophic vaginitis

E) Endometrial cancer

Answer 15.15.1 The correct answer is "C." Polycystic ovarian disease does not cause vaginal bleeding in postmenopausal women. All the other choices are diagnostic considerations in a postmenopausal female with vaginal bleeding or spotting.

Question 15.15.2 Which of the following studies should you consider obtaining?

A) Urinalysis

B) Pap smear

C) Endometrial biopsyv

D) Stool guaiac

E) All of the above

Answer 15.15.2 The correct answer is "E." The patient could easily mistake the source of the bleeding; thus, it is prudent to rule out a rectal or urinary source. Cervical and endometrial evaluations are necessary to rule out gynecological pathology, such as endometrial polyp, hyperplasia, or cancer.

Question 15.15.3 All of the following increase the risk of endometrial cancer EXCEPT:

A) Smoking

B) Obesity

C) Unopposed estrogen

D) Diabetes

E) Hypertension

Answer 15.15.3 The correct answer is "A." Endometrial cancer is believed to be caused by unopposed estrogen stimulation of the endometrium. Smoking decreases luteal phase estrogen and is epidemiologically linked to a decrease in the risk for endometrial carcinoma (of course, this risk is offset by the other adverse health effects of smoking—go figure!). Risk factors and protective factors are listed in the Tables 15-6 and 15-7.

TABLE 15-6aRISK FACTORS FOR ENDOMETRIAL CANCER

View Table||Download (.pdf)

TABLE 15-6 RISK FACTORS FOR ENDOMETRIAL CANCER

Advancing age

Obesitya

Nulliparity

Early menarche

Late menopause

Chronic anovulation (e.g., PCOS)

Unopposed exogenous estrogen use

Tamoxifen

Hypertension

Diabetes

Estrogen producing tumor

Lynch Syndrome

TABLE 15-7aPROTECTIVE FACTORS FOR ENDOMETRIAL CANCERa

You review your patient's test results: normal Pap smear and urinalysis, stool guaiac negative for blood, and endometrial biopsy with fragments of benign polyp.

Over the next 3 years, she continues to have rare occurrences of vaginal spotting. She has declined further evaluation given the infrequency of the episodes. However, over the last several months, she has experienced an increase in the amount and frequency of the bleeding. Other than an interval weight gain of 27 lb, there has been no change in her examination. Repeat endometrial biopsy reveals complex hyperplasia with atypia, and the pathologist cannot rule out endometrial cancer.

Question 15.15.4 Of the following, the most appropriate intervention is:

A) Repeating the endometrial biopsy

B) Performing a transvaginal ultrasound to assess endometrial thickness

C) Starting high-dose progestin therapy

D) Starting high-dose selective estrogen receptor modulator therapy (e.g., tamoxifen)

E) Arranging for definitive management (e.g., hysterectomy)

Answer 15.15.4 The correct answer is "E." Her biopsy findings are highly abnormal and may indicate already existing carcinoma that was not contained in the sample examined. Referral to a gynecologist for definitive management is warranted at this time.

When Mrs Kline is 2 days postpartum The nurse would expect fundal height to be?
HELPFUL TIP:

In postmenopausal women, an endometrial stripe of greater than 5 mm on ultrasound is suggestive of endometrial cancer. With an endometrial stripe of <4 mm, malignancy is rare. In evaluation of postmenopausal women with vaginal bleeding, transvaginal ultrasound is an alternative to endometrial biopsy when an endometrial biopsy cannot be done. Further benefit of obtaining a transvaginal ultrasound, regardless of endometrial biopsy, is that it can detect structural lesions or masses.

Objectives: Did you learn to…

  • Evaluate postmenopausal bleeding?

  • Manage a patient with postmenopausal bleeding?

  • Assess patient for risk of endometrial cancer?

A 15-year-old nulligravid female presents with her mother for evaluation of painful periods. Menarche was at age 14. Her periods are typically every 4 to 8 weeks and are associated with severe cramping. She has missed 1 to 2 days of school with each menses because of pain. She denies intercourse. She has never had a pelvic examination. Her review of systems is otherwise negative.

Question 15.16.1 What is the MOST LIKELY etiology of her irregular cycles?

A) Pregnancy

B) Endometriosis

C) Anovulation

D) Hyperthyroidism

E) Imperforate hymen

Answer 15.16.1 The correct answer is "C." Abnormal uterine bleeding is common among adolescent girls who have reached menarche. The first few years of menstruation are often characterized by irregular cycles as a result of anovulation. Pregnancy and imperforate hymen lead to absence of menses, not irregular menses. While hyperthyroidism may lead to irregular cycles, it does not typically cause dysmenorrhea and is usually associated with other systemic complaints. Additionally hyperthyroidism would be unusual, but not out of the question, in a patient of this age. Endometriosis may cause dysmenorrhea, but is unlikely to occur in a patient this young; most cases of endometriosis present in patients aged 20s to 30s.

Question 15.16.2 What is the etiology of her dysmenorrhea?

A) Prostaglandin release

B) Streptococcal endotoxin release

C) Estrogen release

D) Excessive testosterone production

Answer 15.16.2 The correct answer is "A." Dysmenorrhea is the term that describes excessive pain in association with menstruation. It is the most common gynecologic complaint, affects about half of all adolescent females, and is the leading cause of periodic school absenteeism. The pathogenesis of dysmenorrhea involves excess prostaglandin release, which causes prolonged, painful uterine contractions. It can be divided in two main subtypes: primary and secondary. Primary dysmenorrhea usually starts before the age of 20 and has a tendency to occur with menarche. It is caused by prostaglandin stimulation of the myometrium. Secondary dysmenorrhea typically arises after the age of 20 and is associated with pelvic pathology or other organic disease.

You perform a physical examination, revealing normal vital signs, normal weight, a benign abdomen, Tanner stage V, and no signs of androgen excess.

Question 15.16.3 Which of the following is the best next step in caring for this patient?

A) Offer reassurance and observation

B) Initiate combined hormonal contraception

C) Initiate a GnRH agonist

D) Prescribe a narcotic analgesic

E) Refer for diagnostic laparoscopy

Answer 15.16.3 The correct answer is "B." OCPs offer cycle regulation and a reduction in dysmenorrhea. "A," expectant management, is inappropriate given the severity of symptoms and availability of safe and effective treatment. A further workup is not needed at this stage, as her history is straightforward and her physical examination is reassuring. She certainly does not need surgery now ("E"). "C" is incorrect. GnRH agonists will induce amenorrhea, hot flashes, accelerated bone loss, are expensive, and require add-back estrogen when utilized longer than 6 months. Narcotic analgesics ("D") do not help reduce prostaglandin levels and are not appropriate for pain control in this case. NOTE: Although not listed in the answers, NSAIDs are also quite effective at treating dysmenorrhea and should be considered as a first-line drug. Anecdotal evidence suggests that mefenamic acid (Ponstel) may be more effective for dysmenorrhea than other NSAIDs. Acetaminophen is not as effective as NSAIDs.

When Mrs Kline is 2 days postpartum The nurse would expect fundal height to be?
HELPFUL TIP:

The average age of menarche is 12.8 years in the United States, with the range from 10 to 15 years.

Your patient and her mother opt to try hormonal regulation with birth control pills. She returns for follow-up in 4 months and is doing well. She admits to being sexually active.

Question 15.16.4 In addition to reviewing the use of birth control pills, she should be questioned or counseled about which of the following?

A) Knowledge of sexually transmitted diseases and use of condoms

B) Age of her boyfriend

C) Consensual nature of her relationship

D) HPV vaccination

E) All of the above

Answer 15.16.4 The correct answer is "E." Visits for contraception are great opportunities for you to discuss safe sexual practices with the patient. Such an interview should include evaluation for sexual assault, coercion, or abuse. Although she is a little late for starting HPV vaccination, "D" is still correct (better late than never—truly!). The Advisory Committee on Immunization Practices (ACIP) recommends the routine use of quadrivalant (Gardasil), 9-valent, or bivalent HPV vaccine in females from age 9 to 26 years. The quadrivalent HPV vaccine protects against HPV types 6, 11, 16, and 18. HPV types 6 and 11 are commonly associated with genital warts, while 16 and 18 are commonly responsible for cervical cancer. The 9-valent vaccine protects against HPV types 6, 11, 16, 18, 31, 33, 45, 52, and 58. Both the quadrivalent and the 9-valent vaccines are given in 3 doses (0, 2, and 6 months from first dose). There is also a bivalent vaccine (brand name Cervarix) that protects against HPV types 16 and 18 only and is approved for use in females aged 11 to 25 years. It is given as a three-injection series at 0, 1, and 6 months. In general, the 9-valent vaccine should be used if it is an option.

When Mrs Kline is 2 days postpartum The nurse would expect fundal height to be?
HELPFUL TIP:

HPV is the most common viral sexually transmitted infection in the United States with point prevalence in females ranging from 26% to 64%. Many patients have serial infections with different HPV types. A two-shot series may be as effective as a three-shot series but has not yet been endorsed as official policy. If the vaccination series is interrupted for any length of time, it can be resumed without restarting the series.

When Mrs Kline is 2 days postpartum The nurse would expect fundal height to be?
HELPFUL TIP:

The best time to check a pregnancy test is after the first missed menses. Otherwise, you risk having a false-negative test. Even on day 1 of a missed menses, the sensitivity is only 90% (thus, patients can present with an ectopic pregnancy with a negative urine pregnancy test).

You lose touch with the patient, and she discontinues her OCP. Years later when she returns for a physical examination—and you're still paying off your student loans—she complains of increasing irritability along with intermittent bloating and swelling during the week before her period each month. Although she is annoyed by these symptoms, they are not so severe as to interfere with her usual activities. Her menses now occur monthly without intermenstrual spotting or missed periods.

Question 15.16.5 What is her most likely diagnosis?

A) Major depression

B) Premenstrual dysphoric disorder (PMDD)

C) Premenstrual syndrome (PMS)

D) Polycystic ovary syndrome (PCOS)

E) Hypothyroidism

Answer 15.16.5 The correct answer is "C." PMS is a constellation of physical, emotional, and behavioral symptoms. It is cyclical in nature, occurs during the second half of the menstrual cycle (luteal phase, 7–10 days before menses), and resolves soon after menses. A symptom-free interval occurs during the first half of the cycle (follicular phase). PMDD is more severe but occurs during the same time frame as PMS. Symptoms of PMDD include labile mood, depressed mood, irritability, feelings of hopelessness, hypersomnia or insomnia, and decreased interest in usual activities. PMDD is diagnosed by DSM-V criteria and some functional impairment must be present.

When Mrs Kline is 2 days postpartum The nurse would expect fundal height to be?
HELPFUL TIP:

Premenstrual symptoms exist on a continuum with up to 90% of women affected by minimal PMS symptoms while 10% are severely affected. This group with more severe symptoms can be categorized as having PMDD.

Question 15.16.6 Each of the following is a key element of the diagnosis of PMS EXCEPT:

A) Physical symptoms of bloating, swelling, and/or fatigue

B) Elevated luteinizing hormone to follicle stimulation hormone (LH:FSH) ratio

C) Restriction of symptoms to the luteal phase of the menstrual cycle

D) Exclusion of other diagnoses that may better explain the symptoms

Answer 15.16.6 The correct answer is "B." PMS is a clinical entity and no laboratory data exist to aid in diagnosis. All other options described above are correct. An elevated LH:FSH ratio of 3:1 in the face of appropriate symptoms is suggestive of PCOS.

Question 15.16.7 Each of the following is a possible treatment option for PMS and PMDD EXCEPT:

A) Supportive therapy/counseling

B) Aerobic exercise

C) Selective serotonin reuptake inhibitors

D) Thiazide diuretics

E) Calcium

Answer 15.16.7 The correct answer is "D." Thiazide diuretics are not helpful in PMS or PMDD, but all of the other options are potentially useful. Treatment options that have been shown to help with PMS are listed in Table 15-8.

TABLE 15-8TREATMENT OPTIONS FOR PREMENSTRUAL SYNDROME

View Table||Download (.pdf)

TABLE 15-8 TREATMENT OPTIONS FOR PREMENSTRUAL SYNDROME

Nonpharmacologic

Aerobic exercise

Increased intake of complex carbohydrates and fiber

Reduction in sodium, caffeine, and alcohol intake

Supportive psychotherapy

Pharmacologic

Anxiolytics such as buspirone and benzodiazepines

Calcium and vitamin D supplementation

Danazol

Hormonal treatment (combination OCPs, GnRH agonists, progesterones)

NSAIDs

Serotonergic antidepressants such as SSRIs and venlafaxine

Spironolactone (NOT thiazide diuretics)

Vitamin B6 (pyridoxine)

Chasteberry

Objectives: Did you learn to…

  • Evaluate concerns about menarche and describe normal early menstrual patterns?

  • Evaluate and manage dysmenorrhea?

  • Diagnose and treat PMS?

A frantic 25-year-old patient calls you. She and her boyfriend were having intercourse and the condom broke at the time of ejaculation about 16 hours ago. She does not use any other form of contraception. Her last menstrual period was about 2 weeks ago. You tell her that 8% of women become pregnant after a single act of coitus. She is mortified, exclaiming, "I've always been in the top 8% of everything!"

Question 15.17.1 Appropriate methods of "emergency contraception" for this patient include:

B) Ethinyl estradiol plus levonorgestrel (Yuzpe regimen)

C) High-dose ethinyl estradiol (Ivanapyuk method)

D) A and B only

E) A, B, and C

Answer 15.17.1 The correct answer is "D." Many OCPs are also effective if used at the right doses and within 72 hours (the Yuzpe method). However, as of 2015, only Plan B and ulipristal (an antiprogestin, brand name Ella) are FDA-approved for postcoital contraception. Plan B uses a high dose of levonorgestrel and is more effective than the combined OCPs for postcoital contraception. Plan B is available over the counter (in the United States in 2015). Ulipristal appears to be more effective than Plan B and may be taken up to 120 hours after unprotected intercourse but is only available by prescription. Copper IUD is another option and may be used up to 8 days after unprotected intercourse. "C" is incorrect as estrogen alone is not known to be effective as emergency contraception, and "Ivanapyuk" is a made-up name (but a good description of what happens to a patient who takes a massive dose of estrogen).

When Mrs Kline is 2 days postpartum The nurse would expect fundal height to be?
HELPFUL TIP:

Prescribe an antiemetic with postcoital OCPs as nausea and vomiting are common side effects. Plan B has fewer GI side effects. About the only contraindication to postcoital treatment is active pregnancy. Use a progestin-only regimen in women with a history of thromboembolism.

When Mrs Kline is 2 days postpartum The nurse would expect fundal height to be?
HELPFUL TIP:

Mifepristone (Mifeprex) is not approved in the United States for postcoital pregnancy prophylaxis but is very effective (99–100%) and has a more favorable side-effect profile when compared with other regimens. Mifepristone is approved by the FDA to use in conjunction with misoprostol (Cytotec) for termination of early pregnancy (within 49 days of last menstrual period). Of note, there have been serious adverse events associated with this combination, including sepsis from Clostridium infections and even death. However, these events are very rare.

You recommend Plan B. Since she now trusts you, she schedules an appointment with you for evaluation of intermittent abdominal and pelvic pain. Her pain has gradually worsened over the last 2 years and is almost omnipresent. Now she complains of severe abdominal cramping and stabbing in the right lower quadrant. The pain radiates to the left lower quadrant at times and is worse during menstruation. Her periods have become heavier and occasionally irregular. She has no bowel or bladder symptoms. She has been missing work 1 or 2 days each month and is now concerned about her job.

Your examination reveals a well-developed woman who looks depressed and uncomfortable. Her abdomen is soft, nondistended, and diffusely tender to palpation in the lower quadrants. There is no evidence of guarding, rebound tenderness, or palpable masses. She has no back tenderness. The external genital and vaginal examinations show no lesions or erythema. There is a creamy discharge noted at the cervix. Bimanual reveals a retroverted uterus with uterosacral nodularity palpable. Both adnexal areas are tender to examination, but without masses.

You get a pregnancy test, cultures, and urinalysis, all of which are negative.

Question 15.17.2 Based on her symptoms and your physical examination, what is the most likely etiology of the patient's chronic pelvic pain?

A) Irritable bowel syndrome

B) Myofascial pain disorder

C) Endometriosis

D) Cervical dysplasia

E) Painful bladder syndrome (the disease formerly known as interstitial cystitis)

Answer 15.17.2 The correct answer is "C." There were no bowel symptoms elicited on the history to suggest irritable bowel syndrome. There were no signs elicited on the examination to suggest a myofascial pain disorder. The patient's history and physical examination, including uterosacral nodularity, is consistent with a diagnosis of endometriosis. Dysplasia is typically asymptomatic. There were no bladder symptoms elicited on the history to suggest painful bladder syndrome. Be truthful; have you ever actually felt ligament nodularity?

When Mrs Kline is 2 days postpartum The nurse would expect fundal height to be?
HELPFUL TIP:

If after a careful assessment the diagnosis of endometriosis is highly likely, empiric therapy is considered a viable alternative to laparoscopy and preferred by many experts. However, definitive diagnosis relies upon direct visualization of endometrial implants confirmed by histologic examination.

She does not want laparoscopy. You offer alternatives, and she elects to undergo cycle suppression with a 3-month trial of leuprolide (Lupron) and to complete a pain calendar. You see her in follow-up in 3 months, and she is feeling much better. The pain has been almost completely suppressed, and she has missed only 1 day of work since you last saw her—but that was for a Star Trek convention. After all, the future birthplace of Captain Kirk is located in Iowa (Riverside, Iowa). She has hot flashes, but they are minor.

Question 15.17.3 What is the most appropriate management at this point?

A) Continue the Lupron for up to another 3 months (6 months total)

B) Stop the Lupron and monitor

C) Switch to a trial of cycle suppression using Depo-Provera (medroxyprogesterone acetate) or continuous low-dose OCPs

D) Switch to a trial of Premarin (conjugated estrogens)

E) A or C

Answer 15.17.3 The correct answer is "E." For pain relief, treatment with a GnRH agonist for 3 to 6 months is effective in most patients. Oral contraceptives, as well as oral or depo progestins (Provera), are more effective than placebo. Given the marked treatment success with the depo-Lupron, discontinuing treatment would likely result in recurrence of the patient's pain symptoms. Since endometriosis is estrogen dependent, use of estrogen (Premarin) theoretically could worsen symptoms. Another option is treatment with danazol. However, danazol is less well tolerated than GnRH agonists, Provera, or OCPs.

When Mrs Kline is 2 days postpartum The nurse would expect fundal height to be?
HELPFUL TIP:

NSAIDs are useful monotherapy in patients with mild endometriosis and are useful in combination with hormonal therapy (e.g., OCPs) for patients with more severe symptoms.

The patient is concerned about how endometriosis may affect her future fertility. You recall that she is 25 years old and has never attempted pregnancy (at least not intentionally). She has regular menstrual cycles.

Question 15.17.4 What will you tell her?

A) "You are surely infertile. Look into adoption"

B) "You just cannot know for sure until you have tried to conceive"

C) "What are you worried about? There is no association between endometriosis and infertility"

D) "Look at my face. See how tired I am? Do you really want kids?"

Answer 15.17.4 The correct answer is "B." It is impossible to predict fertility and infertility based on the available information. Early-stage endometriosis is not likely to be associated with alterations in fecundity. If the patient is willing, a diagnostic laparoscopy may aid in visualization of anatomic pathology and allow one to render a guess as to possible tubal disease.

When Mrs Kline is 2 days postpartum The nurse would expect fundal height to be?
HELPFUL TIP:

Chronic pelvic pain is a symptom, not a specific disease. By definition, chronic pelvic pain refers to pain that has been present for more than 6 months and for which a thorough investigation has been negative. Here is a helpful starting guide for elucidating the underlying causes of chronic pelvic pain:

  • Is the pain cyclic? It may be related to endometriosis, dysmenorrhea, adenomyosis, or other diseases that respond to hormones such as irritable bowel syndrome and interstitial cystitis.

  • Is the pain noncyclic? It may be urinary, constipation, a myofascial trigger point in the abdominal wall, etc.

  • Always ask about sexual and physical abuse: There is a high correlation between chronic pelvic pain and a history of sexual abuse.

  • To go by the textbook, the "diagnosis" of chronic pelvic pain requires a negative diagnostic laparoscopy. However, laparoscopy may not be performed in all cases.

Objectives: Did you learn to…

  • Manage patients who desire emergency oral contraception?

  • Define and evaluate chronic pelvic pain?

  • Describe the ramifications, evaluation, diagnosis, and management of endometriosis?

When Mrs Kline is 2 days postpartum The nurse would expect fundal height to be?

A 19-year-old sexually active female presents to your urgent care center with a foul smelling vaginal discharge. She has noted the discharge for about 3 days. On examination, she is in no acute distress, and her vital signs are normal. Her pelvic examination is remarkable for mild vaginal erythema and a frothy gray discharge. You note a malodorous discharge and suspect Trichomonas (bacterial vaginosis can also be malodorous, of course, with a fishy smell). A wet prep confirms your diagnosis.

At this point in time, you recommend that she also be tested for:

A) Chlamydia and gonorrhea

B) Herpes simplex

C) Hepatitis A

D) All of the above

The correct answer is "A." Routine screening for chlamydia and gonorrhea infection is recommended for all sexually active adolescents. Given the presence of one STI, it is appropriate to offer testing for other STIs at this visit. Currently, herpes simplex virus (HSV) is not routinely tested for in asymptomatic persons, and the USPSTF recommends against serologic screening for HSV. Hepatitis A is not a sexually transmitted disease. It may be appropriate to offer testing for hepatitis B, HIV, syphilis, etc., as individual cases dictate.

When Mrs Kline is 2 days postpartum The nurse would expect fundal height to be?

An 80-year-old woman presents for evaluation of a "bulge" she noted after gardening over the weekend … or maybe it's been there for years—she's not sure. She has no discomfort and no difficulty with bowel or bladder elimination. On examination, you note her cervix extends 1 cm beyond the vaginal introitus with Valsalva. There are no lesions or excoriations noted.

Of the following, what is the best initial treatment option?

A) Hysterectomy

B) Trachelectomy

C) Pessary trial

D) Bed rest

E) Hormone therapy

The correct answer is "C." Seventy percent of women who are fitted with a pessary are satisfied at 5-year follow-up. Hysterectomy and trachelectomy (removal of the cervix) are both unnecessarily invasive treatments without trying a conservative strategy.

When Mrs Kline is 2 days postpartum The nurse would expect fundal height to be?

What is the best way to manage an asymptomatic 4-cm ovarian cystic mass found initially on pelvic examination and confirmed by pelvic ultrasound as a simple cyst in a 22-year-old female who is otherwise healthy?

A) Referral to a gynecologist

B) Start hormonal therapy to reduce ovulation

C) Expectant management with repeat ultrasound in 2 months

D) Serum CA-125 level

The correct answer is "C." A 4-cm ovarian mass likely represents a functional cyst in a woman who is cycling (reproductive age). In general, premenopausal women with simple cysts 10 cm or smaller may be managed conservatively with serial ultrasonography in 4 to 12 weeks. Since the mass is asymptomatic and without concerning ultrasound findings (complex mass, septations, solid components), expectant management is the best option. No further evaluation is warranted at this time. CA-125 is a tumor marker for epithelial ovarian cancer, but it is not useful as a screening test.

When Mrs Kline is 2 days postpartum The nurse would expect fundal height to be?

What is the best first step in managing an asymptomatic palpable 4 cm adnexal mass in a 76-year-old postmenopausal woman?

A) Referral to a gynecologist

B) Start hormonal therapy to suppress FSH and LH

C) Expectant management with repeat examination in 2 months

D) Serum CA-125 level

E) Pelvic ultrasound

The correct answer is "E." Unlike a relatively small palpable ovarian mass in a reproductive-age woman, a palpable ovarian mass in a postmenopausal woman represents ovarian malignancy until proven otherwise. The best initial imaging study for evaluation of a pelvic mass is ultrasound. Ultrasound will not only identify the location of the mass but will also identify its internal consistency. Characteristics suggestive of cancer include bilaterality, solid and cystic components, thick septations, and the presence of ascites. CA-125 is a marker for epithelial ovarian cancer and may assist in evaluation, but it cannot be relied upon to rule in or to rule out cancer as a diagnosis. (Best case scenario, in appropriately selected patients, CA-125 has a sensitivity around 95% and a specificity around 90% for the diagnosis of ovarian cancer.) CA-125 is useful in follow-up of patients with a history of ovarian cancer.

When Mrs Kline is 2 days postpartum The nurse would expect fundal height to be?

What is the leading cause of death from a gynecologic malignancy in American women?

A) Ovarian cancer

B) Uterine cancer

C) Cervical cancer

D) Fallopian tube cancer

E) Vaginal cancer

The correct answer is "A." Ovarian cancer is the leading cause of death from gynecologic malignancy, is the second most common gynecologic malignancy, and is the fourth leading cause of cancer death in women. Endometrial cancer is the most common gynecologic malignancy but also one of the most treatable. Cervical cancer is the third most common gynecologic malignancy. Both fallopian tube and vaginal malignancy are relatively rare.

When Mrs Kline is 2 days postpartum The nurse would expect fundal height to be?

How does ovarian cancer typically present?

A) Early satiety

B) Abdominal fullness and pain

C) Urinary obstruction

D) Asymptomatic mass noted on routine examination

E) A and B

The correct answer is "E." There are no specific early symptoms of ovarian cancer. Thus, most patients present with symptoms associated with increasing tumor mass: early satiety, abdominal fullness or bloating, and abdominal pain. Unfortunately, ovarian cancer is rarely identified at an early stage on routine annual examination.

When Mrs Kline is 2 days postpartum The nurse would expect fundal height to be?
HELPFUL TIP:

An ovarian mass is more likely to be malignant if the patient is premenarcheal or postmenopausal, is greater than 10 cm in diameter, and has solid or complex cystic features on ultrasound.

While covering the emergency department on the graveyard shift a 21-year-old college student presents sobbing with a friend. Her friend says, "She's been raped."

Question 15.18.1 Relevant history includes all of the following EXCEPT:

A) Whether force was used and what type

B) Physical characteristics of the assailant

C) Details regarding penetration (vaginal, anal, oral)

D) Number of sexual partners the victim has had in her lifetime

E) Condom use

Answer 15.18.1 The correct answer is "D." The patient's past sexual history is not relevant in the evaluation of sexual assault. The other issues are pertinent to the case. Although it may be difficult for the patient to relive the experience, you should try to obtain a detailed history of the assault. In order to assess her risk for pregnancy and infection, you need to ask about the area penetrated (e.g., vaginal, oral, or anal penetration), whether the assailant ejaculated, and if a condom was used. In a sexual assault case, your job is also to collect evidence, including pertinent historical elements (e.g., number of assailants, names, physical appearance, whether force was used, and what type—threat, restraints, weapons, etc.).

When Mrs Kline is 2 days postpartum The nurse would expect fundal height to be?
HELPFUL TIP:

Sexual assault includes genital, anal, or oral penetration by a part of the assailant's body or by an object. By definition, it occurs without the victim's consent and need not involve direct force or violence.

Question 15.18.2 Which of the following are important physical elements to collect for the forensic evaluation in this case?

A) Combed specimens from the scalp and pubic hair

B) Swabs of the oral, vaginal, and rectal mucosa

C) The patient's clothing

D) Fingernail scrapings

E) All of the above

Answer 15.18.2 The correct answer is "E." All of the items listed will be important to the investigation. Evidence collection kits for sexual assault cases ("rape kits") should be available in your emergency department.

Although apparently inebriated, the patient is able to give a coherent history. When you broach the subject of physical examination, her friend says, "Look, she was raped an hour ago. Can't you let her just recover a bit before you violate her all over again?"

Question 15.18.3 Which of the following is the most appropriate response?

A) "Of course. Come back tomorrow after you have sobered up and taken a shower"

B) "An examination is important for your health and in the event that this becomes a criminal case. The yield of the examination declines with time. Even if you don't feel like prosecuting now, you may decide to do so in the future, and the best evidence is gathered early"

C) "The examination has a fairly high yield even a week after the assault, so take your time on this"

D) "Under federal law I am required to perform this examination"

Answer 15.18.3 The correct answer is "B." The yield of a forensic examination declines with time. Even if a patient states that she does not want to prosecute the assailant, she should be encouraged to have the examination done in case she changes her mind. Also, you are concerned about her health, and she may be at risk for sexually transmitted diseases, pregnancy, and traumatic injury. Despite the fact that yield does decline with time, reliable evidence may still be gathered up to 5 days after the assault.

When Mrs Kline is 2 days postpartum The nurse would expect fundal height to be?
HELPFUL TIP:

"Rape trauma syndrome" generally occurs in three stages. The first includes anger, anxiety, guilt, shame, sleep disturbance, etc. The second stage includes somatic complaints (pelvic pain, other pain) and psychiatric complaints (depression, phobias, etc.). Some patients will resolve these issues while others will develop post-traumatic stress syndrome. The third stage is renormalization.

Question 15.18.4 Which of the following is the LEAST appropriate to offer this patient at this point in time?

A) HIV antigen/antibody testing

B) HSV antibody testing

C) Prophylactic treatment for gonorrhea and chlamydia

D) Mental health services referral

E) Emergency contraception

Answer 15.18.4 The correct answer is "B." Herpes virus antibody testing will only tell you if she has been exposed to HSV in the past. Further recommendations include syphilis testing, hepatitis B antibody testing, performing a wet prep of a vaginal sample, and checking a urine pregnancy test.

Objectives: Did you learn to…

  • Evaluate a patient for sexual assault?

  • Manage a patient who has been the victim of sexual assault?

A 21-year-old woman presents to your office complaining of pelvic pain with intercourse, worse over the last 2 weeks. She also complains of not getting pregnant, even though she's had several partners over her last 3 years of sexual activity and has been trying to get pregnant with the same partner for the past 6 months. She states she never has used birth control of any type—not even once. You commend her on commitment to her principles. She started her periods around age 14 but has only had a couple of periods since then. Apparently, this pattern of menstruation is normal for her family, as her mother was the same way.

On physical examination, you notice the patient is a centrally obese young woman, afebrile, with (culturally defined) excess hair noted down the side of her face and under her chin. She also has some erythematous pustules on her cheeks.

Question 15.19.1 Which of the following lab results would be most consistent with the history and examination findings?

A) Positive urine pregnancy test

B) Low TSH level

C) Elevated CA-125 level

D) Mildly elevated androgens

E) Prolactin level more than three times normal

Answer 15.19.1 The correct answer is "D." This patient gives a history and has an appearance consistent with polycystic ovarian syndrome (PCOS). The clinical features of PCOS include oligomenorrhea (90%), hirsutism (80%), obesity (50%), amenorrhea (40%), and infertility (40%). Early symptoms in an adolescent may consist only of irregular periods, acne, and central obesity. Clinical or laboratory evidence of androgen excess may be present, such as mildly elevated testosterone. Note that depending on the diagnostic criteria you are using for PCOS, an ultrasound may not be necessary. An LH:FSH ratio greater than 3:1 adds further support to the diagnosis. You should certainly do a pregnancy test, a TSH, and a prolactin level. However, this patient most likely has PCOS.

When Mrs Kline is 2 days postpartum The nurse would expect fundal height to be?
HELPFUL TIP:

The most recent evidence points to insulin resistance as the underlying cause of PCOS, and these patients may have acanthosis nigricans. Insulin resistance can be quantified by calculating the ratio of fasting glucose to insulin. A ratio of less than 4.5 indicates insulin resistance. Insulin resistance stimulates ovarian androgen production, which leads to anovulation.

You proceed with the pelvic portion of the examination, noting the patient also has a diamond-shaped, rather than triangular-shaped, pubic hair pattern. You find no lesions on the vulva or in the vagina. However, the cervix appears reddened, with an almost strawberry texture. And, even though there is a generous amount of yellowish, malodorous leukorrhea in the vaginal vault, there is no notable pus at the cervical os. Bimanual examination is limited due to the patient's obesity.

Question 15.19.2 Of the following, which is the most likely cause of her cervicitis?

A) HSV infection

B) Trichomonas vaginalis infection

C) Candida albicans infection

D) PID

E) Bacterial vaginosis

Answer 15.19.2 The correct answer is "B." Trichomonas is a protozoan that is sexually transmitted and can cause urethritis in both sexes. However, in women, it most commonly causes ulceration of the cervical mucosa with punctate hemorrhages known as a "strawberry cervix." Signs and symptoms also include a malodorous discharge and occasional vulvar and vaginal irritation. The cervix can be somewhat tender to touch, either during examination or intercourse, and patients often complain of a nonspecific pelvic pain. Males are often asymptomatic.

The wet mount demonstrates Trichomonas; there is no evidence of yeast or clue cells. You send samples for chlamydia and gonorrhea tests as well as a Pap smear. You recommend testing for HIV, syphilis, and hepatitis B, and she agrees.

Question 15.19.3 For her Trichomonas vaginal infection, you prescribe:

A) Flagyl (metronidazole) 2 g orally in a single dose

B) MetroGel-Vaginal (topical vaginal metronidazole) 5 g applied nightly for 5 days

C) Diflucan (fluconazole) 150 mg orally in a single dose

D) Zithromax (azithromycin) 1 g orally in a single dose

E) Levaquin (levofloxacin) 250 mg orally in a single dose

Answer 15.19.3 The correct answer is "A." The best choice is oral metronidazole. Topical antibiotic gels, creams, or ovules—either metronidazole or clindamycin (Cleocin)—only treat bacterial vaginosis, as the concentration is insufficient to reach the protozoa in the glands and urethral areas. The remaining options are all incorrect for treating Trichomonas. See Table 15-9 for more on diagnosis and treatment of infectious vaginitis.

TABLE 15-9VAGINITIS DIAGNOSIS AND TREATMENT

View Table||Download (.pdf)

TABLE 15-9 VAGINITIS DIAGNOSIS AND TREATMENT

OrganismDischargeOdorMicroscopypHTreatment
Bacterial vaginosis Thin, gray, homogeneous Fishy with positive "whiff test" Clue cells >4.5 Metronidazole 500 mg BID × 7 days or clindamycin 300 mg BID × 7 days or topical metronidazole 1 applicatorful intravaginally daily × 5 days (lower success rate)
Candida Adherent, white, "cottage cheese" like Neutral Pseudohyphae but only 65–85% sensitive <4.5 Fluconazole oral, topical clotrimazole, miconazole, etc.
Trichomonas Copious yellow, gray, green, foamy. Friable "strawberry" cervix Malodorous Trichomonads >4.5

Metronidazole 2 g PO once (recommended), or 500 mg

BID × 7 days (alternative)

When Mrs Kline is 2 days postpartum The nurse would expect fundal height to be?
HELPFUL TIP:

As with other STIs, a patient with Trichomonas should have her partner tested and treated (or just treated depending whether or not this is allowable under your state law).

When Mrs Kline is 2 days postpartum The nurse would expect fundal height to be?
HELPFUL TIP:

While it makes sense that single-dose azithromycin would work better in treating Chlamydial cervicitis because of compliance issues, the cure rate is the same whether azithromycin or the doxycycline is used. There is about a 3% failure rate with azithromycin which isn't seen with doxycycline.

You now return your attention to her PCOS (remember, way back then, the reason she came in?). Her lab results demonstrated a LH:FSH ratio >3; normal TSH and prolactin; slightly elevated testosterone, but still well below the normal male range; fasting glucose:insulin ratio <4.5; and slightly elevated total cholesterol and triglycerides. Her LH:FSH ratio was consistent with the diagnosis of PCOS. She has not menstruated for 4 months. A urine hCG is negative, and a repeat fasting glucose is 120 mg/dL.

Question 15.19.4 Which of the following recommendations should you make now?

B) Attempt weight loss through a nutritious diet and increased exercise

C) Initiate oral contraceptives to regulate menses

D) A and C

E) All of the above

Answer 15.19.4 The correct answer is "E." This patient has glucose intolerance (elevated fasting glucose and ratio of glucose: insulin <4.5), and it is reasonable to initiate dietary and medical therapy at this point in time. Another option is to start with lifestyle modifications and check fasting glucose again 3 to 6 months later. Due to increased risk of endometrial carcinoma in patients who have rare menses, it is important to regulate her cycles. OCPs ("C") can accomplish menstrual regulation.

When Mrs Kline is 2 days postpartum The nurse would expect fundal height to be?
HELPFUL TIP:

For hirsutism associated with PCOS, spironolactone is usually first-line therapy unless the patient has a contraindication; traditional hair removal techniques will still be required for the existing hair growth. Since spironolactone can result in feminization of a male fetus, patients taking spironolactone must be using reliable birth control.

When Mrs Kline is 2 days postpartum The nurse would expect fundal height to be?
HELPFUL TIP:

Not all women with PCOS are obese and hirsute. Many patients may be thin with sparse body hair and present with menstrual irregularities and fertility concerns.

Objectives: Did you learn to…

  • Identify the clinical presentation of PCOS?

  • Diagnose and treat Trichomonas infection?

  • Diagnose and manage PCOS?

When Mrs Kline is 2 days postpartum The nurse would expect fundal height to be?

A 16-year-old female presents with her mother. They don't look happy. Her mother says, "She needs a Pap smear because she's been having sex with a couple of boys—in my house, I will have you know—for a year!" The patient rolls her eyes.

Consistent with published guidelines, you recommend:

A) Pap smear

B) Gonorrhea and chlamydia testing

C) Pap smear and gonorrhea and chlamydia testing

D) Return for a Pap after sexually active for 3 years (age 18 for this patient)

E) A chastity belt

The correct answer is "B." It is now recommended to delay cervical cancer screening until age 21, even if the woman has been sexually active. The reasoning: although adolescent females are frequently infected with HPV, they also easily clear these infections, with 95% of lesions spontaneously regressing. Exceptions to this rule include patients who are immunocompromised (e.g., organ transplant, HIV infection). For this patient, ensuring that she has completed her HPV serious and counseling on safe sexual behavior is also warranted.

When Mrs Kline is 2 days postpartum The nurse would expect fundal height to be?

A 65-year-old female presents for a health maintenance examination. She complains of a vulvar itching due to what she calls "recurrent yeast infections," and her symptoms have worsened over the last few months. She is sexually active with her husband and has experienced dyspareunia with penetration lately. She always uses a water-based lubricant with intercourse. On examination, you find complete loss of the borders of the labia minora, constriction of the vaginal outlet, and several thin white plaques (like parchment paper) on the vulva. There is no other skin or mucosal involvement.

What is the most likely diagnosis?

A) Lichen planus

B) Lichen simplex

C) Lichen sclerosus

D) Vulvovaginal candidiasis

E) Squamous carcinoma

The correct answer is "C." The clinical description above is characteristic of lichen sclerosus, which occurs more commonly in older women but also has a peak in young girls. Almost all lichen sclerosus is intensely pruritic. As lichen sclerosus progresses, there may be loss of labial architecture, stenosis of the introitus, and obliteration of the clitoris. The lesions are usually multiple and appear as thin, shiny, white, wrinkled patches or plaques. The rest are incorrect. However, "E," squamous carcinoma, occasionally can be confused for lichen sclerosus but is more likely to present with ulceration and induration and is less common (though if you are suspicious of cancer, a biopsy is indicated). Patients with lichen sclerosus have a squamous cell cancer risk of 3% to 7% and should have an examination every 6 to 12 months with biopsies as indicated for persistent and nonhealing lesions.

When Mrs Kline is 2 days postpartum The nurse would expect fundal height to be?
HELPFUL TIP:

Initial treatment for lichen sclerosus involves local steroid ointment application. High-potency steroids are initiated and then tapered to the lowest effective potency and frequency that maintain symptom control. Testosterone creams have fallen out of favor due to lesser efficacy and secondary virilization. Vaginal dilators can be used if there is constriction of the vaginal opening causing dyspareunia.

When Mrs Kline is 2 days postpartum The nurse would expect fundal height to be?

A 48-year-old perimenopausal female presents with a 3-day history of vulvar pruritus. Her history is significant for mitral valve replacement and she is on warfarin with INR 2–3. A limited vulvar and vaginal examination reveals significant erythema with satellite lesions on the labia majora. Wet prep microscopy reveals abundant pseudohyphae and inflammatory cells. You somehow assemble all these clues into a diagnosis of candidal vulvovaginitis (and you can tell the patient is impressed when she says, "That's right, genius. I've got a yeast infection"). She enquires about use of oral therapy, as vaginal creams are "messy."

How will you counsel this patient regarding use of oral fluconazole (Diflucan)?

A) "You have no contraindications to oral fluconazole"

B) "Given your use of warfarin, you should not use oral fluconazole"

C) "You will need to stop warfarin while taking oral fluconazole"

D) "You should take extra warfarin if you take oral fluconazole"

The correct answer is "B." There are numerous drug interactions with oral fluconazole, including warfarin (both inhibit CYP3A4). The INR will increase after even one dose of fluconazole therapy. Similarly, the ubiquitous statins are affected by oral fluconazole with several case reports of rhabdomyolysis in the literature. In this case, stick with topical antifungals, although your patient satisfaction scores may plummet. (Incidentally, patient satisfaction has been inversely correlated with healthcare quality—although there appears to be a direct link between increased satisfaction and increased health care expenditure.)

A 27-year-old female presents with her husband seeking advice regarding pregnancy loss. She recently had a miscarriage. Your patient states that this was her third miscarriage in the last 2 years. All three occurred at about 9 weeks of gestation.

Question 15.20.1 Possible explanations for recurrent pregnancy loss in this patient include each of the following EXCEPT:

A) Parental structural chromosome abnormalities

B) Uterine anatomic abnormalities

C) Anticardiolipin antibody syndrome

D) Idiopathic (unexplained etiology)

E) Conception while on oral contraceptives

Answer 15.20.1 The correct answer is "E." Conception while on oral contraceptives will not increase the risk of recurrent spontaneous miscarriages. Parental structural chromosome abnormalities (balanced structural chromosome rearrangement in one partner) are responsible for pregnancy loss in 2% to 4% of couples. Uterine anatomic abnormalities ("B") have been associated with 10% to 15% of pregnancy loss. "D" is true. The majority of couples with recurrent pregnancy loss will have an uncertain etiology despite extensive evaluation (>50%).

When Mrs Kline is 2 days postpartum The nurse would expect fundal height to be?
HELPFUL TIP:

Recurrent pregnancy loss is classically defined as loss of three or more consecutive pregnancies.

The couple desires testing for possible causes of the pregnancy losses.

Question 15.20.2 Of the following, which test(s) should be included in the evaluation?

A) Cultures for bacteria

B) Test for glucose intolerance

C) Maternal antipaternal antibodies

D) Lupus anticoagulant and anticardiolipin antibody

E) All of the above

Answer 15.20.2 The correct answer is "D." Antiphospholipid syndrome is associated with pregnancy loss in 3% to 15% of women with recurrent pregnancy loss. The others are not useful. However, chromosomal testing of the parents (not given as an option in this question) may be useful. Other tests that can be considered include screening for thyroid abnormalities with anti-thyroid peroxidase (TPO) and TSH with free T4.

Evaluation of the recurrent pregnancy loss fails to identify a cause. Thus, like most couples with recurrent pregnancy loss, the etiology remains unexplained.

Question 15.20.3 What is the likelihood that this couple will have a successful pregnancy outcome in the next pregnancy?

A) Highly unlikely, they should consider adoption

B) Less than one in four chances of successful pregnancy

C) 60% to 70% chance of successful next pregnancy

D) You cannot hazard a guess. Amazingly, this has not been studied

Answer 15.20.3 The correct answer is "C." Studies suggest that 60% to 70% of couples with unexplained recurrent pregnancy loss will have a successful next pregnancy.

Objectives: Did you learn to…

  • Define recurrent pregnancy loss and discuss some of its epidemiologic aspects?

  • Enumerate potential causes of recurrent pregnancy loss?

  • Identify etiologies and the workup of recurrent pregnancy loss?

When Mrs Kline is 2 days postpartum The nurse would expect fundal height to be?

A 28-year-old primigravida female presents for an initial obstetric visit. Pelvic examination is consistent with a 6- to 8-week gestation uterus, and the remainder of the examination is unremarkable. As this is her first pregnancy, she has a number of questions. She wants to know how much weight gain is expected and whether she should "watch her weight."

You calculate her BMI as 24 kg/m2 and recommend the following:

A) "Eat anything you want. You're eating for two!"

B) "Your BMI is normal. Your goal is to gain no more than 20 lb"

C) "Your BMI is low. Your goal is to gain 40 lb"

D) "Your BMI is high. Your goal is to gain no more than 15 lb"

E) "Your BMI is normal. Your goal is to gain 30 lb"

The correct answer is "E." The Institute of Medicine recommends weight gain in pregnancy based on pregravid BMI (see Table 15-10). In this patient, her BMI is in the normal range, so her goal for weight gain in pregnancy is 25 to 35 lb. Women with a normal prepregnancy BMI should gain about 1 lb per week during their second and third trimesters.

TABLE 15-10PREGNANCY WEIGHT GAIN GUIDELINES

View Table||Download (.pdf)

TABLE 15-10 PREGNANCY WEIGHT GAIN GUIDELINES

BMI (kg/m2)Goal Weight Gain (kg)Goal Weight Gain (lb)
<18.5 12.5–18.0 28–40
18.5–24.9 11.5–16.0 25–35
25.0–29.9 7.0–11.5 15–25
>30 5–9 11–20

When Mrs Kline is 2 days postpartum The nurse would expect fundal height to be?

A 32-year-old female is coming to see you for amenorrhea. She had regular menses until the last year when they became irregular. She has not had any menses for the past 6 months. She is somewhat distraught because she wants to have a family. Being the smart doctor that you are, you know that a pregnancy test is the first thing to do: it is negative. Being the smart doctor that you are, you also think about the female athlete triad: amenorrhea, an eating disorder, and osteoporosis. She does not meet this profile. You take more of a history and find out the following:

  • There is no additional stress in her life (such as starting college or a new job), weight loss, or illness, etc., which might lead to hypothalamic amenorrhea.

  • She has no galactorrhea (prolactinoma).

  • She has no hot flashes, vaginal dryness, etc. (premature menopause).

  • She denies headaches, visual changes, fatigue, polydipsia, or polyuria (pituitary problems).

  • She has no acne, hirsutism, etc., suggestive of PCOS.

Hey, did you notice how we are cleverly giving you the workup of secondary amenorrhea in the question?

As recommended, you check a TSH and prolactin; they are normal.

The next step in the evaluation of this patient's amenorrhea is:

A) A progestin challenge

B) Hysterosalpingogram to prove cervical patency

C) A trial of oral contraceptives to prove cervical patency

D) An LH to rule out menopause

E) All of the above

The correct answer is "A." A progestin challenge should cause a withdrawal bleed if there is still adequate estrogen (thus ruling out premature ovarian failure). The step following this would be to start a combination oral contraceptive. Failure to induce menses with a combination OCP suggests a mechanical blockage such as Asherman syndrome (scarring of the endometrial lining with intrauterine adhesions). Failure to induce menses with a combination OCP can be followed by a hysterosalpingogram or hysteroscopy to identify any mechanical problems.

You did it! You successfully worked up secondary amenorrhea.

When Mrs Kline is 2 days postpartum The nurse would expect fundal height to be?
HELPFUL TIP:

But wait, there is more! Check an FSH as well. If this is high, it might indicate ovarian failure. If the FSH is normal or low in the absence of circulating estrogen (a negative progestin challenge), consider a pituitary cause of amenorrhea including possible hypothalamic–pituitary axis dysfunction from weight loss, stress, pituitary tumor, empty sella syndrome, etc. No pituitary? No FSH.

Clinical Pearls

  • Do not perform elective deliveries (C-sections or inductions) that are not medically indicated prior to 39 weeks of gestation.

  • Do not perform routine screening for ovarian cancer in asymptomatic, low-to-average risk women.

  • Do not require a Pap and a pelvic examination to prescribe contraception. The only necessary requirement is to perform a detailed history to ascertain risk factors of using hormonal contraception, blood pressure check, and the performance of a urine pregnancy test to ensure the woman is not pregnant.

  • Do not routinely perform episiotomies; there is no evidence that episiotomies reduce perineal trauma, postpartum dyspareunia, etc.

  • Do not start Pap smear screening before the age of 21 regardless of sexual activity status.

  • Due to the increased risk of endometrial cancer, do not use unopposed estrogen in a woman unless she has undergone a hysterectomy.

  • In a pregnant woman with third-trimester vaginal bleeding, do not perform a digital or speculum vaginal examination until you can rule out the diagnosis of placenta previa.

  • Perform routine GBS screening on pregnant women between 35 and 37 weeks of gestation UNLESS the woman has a documented history of GBS bacteriuria during current pregnancy or prior history of infant born with invasive GBS infection, in which cases they would be treated anyway.

  • Recommend folic acid in all women of childbearing age to prevent any risk of neural tube defects if she should become pregnant.

  • Use the ASCCP guidelines to determine frequency of Pap smears and management of abnormal smears.

American College of Obstetricians and Gynecologists. ACOG Practice Bulletin no. 83: Management of adnexal masses. Obstet Gynecol. 2007;110(1):201–214.  [PubMed: 17601923]

American College of Obstetricians and Gynecologists. ACOG Practice Bulletin no. 106: Intrapartum fetal heart rate monitoring, nomenclature, interpretation and general management principles. Obstet Gynecol. 2009;114(1):192–202.  [PubMed: 19546798]

American College of Obstetricians and Gynecologists. ACOG Practice Bulletin No. 112: Emergency contraception. Obstet Gynecol. 2010;115:1100–1109.
[PubMed: 20410799]  

American College of Obstetricians and Gynecologists. ACOG Practice Bulletin No. 130: Prediction and prevention of preterm birth. Obstet Gynecol. 2012;120(4):964–973.
[PubMed: 22996126]  

American College of Obstetricians and Gynecologists. Practice bulletin No. 137: gestational diabetes mellitus. Obstet Gynecol. 2013;122(2, part 1):406–416.
[PubMed: 23969827]  

American College of Obstetricians and Gynecologists. Executive summary: hypertension in pregnancy. Obstet Gynecol. 2013;122(5):1122–1131.  [PubMed: 24150027]

American College of Obstetricians and Gynecologists. Method for estimating due date. Committee Opinion No. 611. Obstet Gynecol. 2014;124:863–866.
[PubMed: 25244460]  

American College of Obstetricians and Gynecologists. Guidelines for acute and emergent therapy for acute-onset severe hypertension during pregnancy and the postpartum period. Am Obstet Gyencol. 2015;No 623:1–5.

Biggs  W, Demuth  RH. Premenstrual syndrome and premenstrual dysphoric disorder. Am Fam Physician. 2011;84(8):918–924.
[PubMed: 22010771]  

Centers for Disease Control (CDC). 2015 Sexually Transmitted Diseases Treatment Guidelines: Gonococcal infections.
[PubMed: 26602616]

Crossman  SH. The challenge of pelvic inflammatory disease. Am Fam Physician. 2006;73:859–864.
[PubMed: 16529095]  

Division of Reproductive Health, National Center for Chronic Disease Prevention and Health Promotion, Centers for Disease Control and Prevention (CDC). U.S. Selected Practice Recommendations for Contraceptive Use, 2013: adapted from the World Health Organization selected practice recommendations for contraceptive use, 2nd edition. MMWR Recomm Rep. 2013;62(RR-05):1–60.

Gradison  M. Pelvic inflammatory disease. Am Fam Physician. 2012;85(8):791–796.
[PubMed: 22534388]  

Hainer  BL, Gibson  MV. Vaginitis: diagnosis and treatment. Am Fam Physician. 2011;83(7):807–815.
[PubMed: 21524046]  

Hatcher  RA,  et al. A Pocket Guide to Managing Contraception. 6th ed. New York: Ardent Media, Inc.; 2005.

Kirkham  C,  et al. Evidence-based prenatal care: Part I. general prenatal care and counseling issues. Am Fam Physician. 2005;71(5):1307–1316.
[PubMed: 15832534]  

Kirkham  C,  et al. Evidence-based prenatal care: Part II. third-trimester care and prevention of infectious diseases. Am Fam Physician. 2005;71:1555–1562.
[PubMed: 15864896]  

Klein  DA,  et al. Provision of contraception: key recommendations from the CDC. Am Fam Physician. 2015;91(9):625–633.
[PubMed: 25955737]  

LeFevre  ML. Low-dose aspirin use for the prevention of morbidity and mortality from preeclampsia: US Preventative Services Task Force recommendation statement. Ann Intern Med. 2014;161:819–826.
[PubMed: 25200125]  

Magnotti  M, Futterweit  W. Obesity and the polycystic ovary syndrome. Med Clin North Am. 2007;91(6):1151–1168.
[PubMed: 17964914]  

Massad  LS,  et al. American Society for Colposcopy and Cervical Pathology 2012 updated consensus guidelines for the management of abnormal cervical cancer screening tests and precursors. J Low Genit Tract Dis. 2013;17(5):S1–S27.  [PubMed: 23519301]

Ott  MA,  et al. Policy statement: Contraception for adolescents. Pediatrics. 2014;134(4):e1244–e1256.  [PubMed: 25266430]

Raina  R,  et al. Female sexual dysfunction: classification, pathophysiology, and management. Fertil Steril. 2007;88(5):1273–1284.  [PubMed: 17991514]

Tan  PC,  et al. Effect of coitus at term on length of gestation, induction of labor, and mode of delivery. Obstet Gynecol. 2006;108(1):134–140.
[PubMed: 16816067]  

Tenore  JL. Methods for cervical ripening and induction of labor. Am Fam Physician. 2003;67:2123–2128.
[PubMed: 12776961]  

Workowski  KA, Bolan  GA. Sexually transmitted diseases treatment guidelines, 2015. MMWR Recomm Rep. 2015;64(RR-3):1–137.
[PubMed: 26042815]  

Zolotor  AJ. Update on prenatal care. Am Fam Physician. 2014;89(3):199–208.
[PubMed: 24506122]  


Where should the fundus be 2 days postpartum?

At about an hour after childbirth, your fundus should be around your belly button (where it was at 20 weeks). After that, it should steadily decrease 1 centimeter every 24 hours. At about one week postpartum, your fundus should be at your pubic bone (where it was at 12 weeks).

Where is fundus 2 weeks postpartum?

By approximately one hour post delivery, the fundus is firm and at the level of the umbilicus. The fundus continues to descend into the pelvis at the rate of approximately one centimeter (finger-breadth) per day and should be nonpalpable by two weeks postpartum.

Where should the fundus be 48 hours after birth?

Therefore, 24 hours after birth the fundus should 1 cm (or one finger breadth) BELOW the belly button…… 48 hours it should be 2 cm below the belly button. At 7 days the fundus should be at the symphysis pubis.

How should the fundus feel 12 hours after birth?

12 hours after delivery, the fundus of the uterus should be firm - we always like firm - midline, meaning in the middle of the body, not deviated to one side or the other, and approximately at the level of the umbilicus, so at the level of the belly button.